Invited Experts on Gaza Jurisdiction Question

George Fletcher Avatar Image Professor George P. Fletcher Cardozo Professor of Jurisprudence Columbia Law School

No Jurisdictional Basis for an Investigation Pursuant to the Palestinian Declaration

The wording of the ICC basic documents, the diplomatic practice of the ICC Rome Conference, Preparatory Commission, and Assembly of States Parties, and scholarly commentary make it clear that the Palestinian Authority is not a “State.”

Summary

The Palestinian Authority is not a state within the meaning of Article 12(3) of the Rome Statute and therefore its submission does not establish the necessary preconditions for the exercise of legal jurisdiction by the Court.

Not only is the Palestinian Authority not recognized as a state by the United Nations, but the ICC’s own documents and practices have not recognized it as a state. When it has been invited to participate, it has been as an “entity” or “other organization” invited to observe. Indeed the underlying question of a two-state solution to the Israeli-Palestinian conflict remains a central issue in the larger diplomatic negotiations and it is not the role of international courts, including the ICC, to undertake a unilateral decision about such a matter.

Argument

The “Declaration recognizing the Jurisdiction of the International Criminal Court”1 submitted by the Palestinian Authority on January 22, 2009, seeks to satisfy the statutory preconditions to the Court’s exercise of jurisdiction in order to investigate and prosecute acts “committed on the territory of Palestine since 1 July 2002”. The declaration refers to Article 12(3) of the ICC Statute as the claimed basis for establishing jurisdiction.2

However, the declaration does not satisfy the requirements of Article 12(3) of the ICC Statute as written, interpreted or applied. Nor does it comport with relevant principles of public international law. It is, therefore, without jurisdictional effect under the Statute.

It is evident, of course, that the declaration cannot change the limitations on jurisdiction that are prescribed by the ICC Statute itself, as adopted on 17 July 1998 by the States assembled at the Diplomatic Conference of Plenipotentiaries in Rome.

The language of Article 12 of the ICC Statute sets forth plain and irreducible “[p]reconditions to the exercise of jurisdiction” by the Court. It states unequivocally that the acceptance of the Court’s jurisdiction is limited to “States” both under Article 12(2) and Article 12(3). However, the Palestinian Authority does not qualify as a “State” or “State Party” under the ICC Statute. It is therefore ineligible, inter alia, to make a declaration under Article 12(3). The language of Article 12(3) was finalized at the Rome conference in July 1998, and has not been the subject of any corrigenda.3

The Palestinian Authority was not a participating “State” at the Rome Conference in 1998. The conference’s official roster of “Participating States” includes 163 States from around the world, but it does not include the Palestinian Authority or “Palestine” as a State. Rather, the Palestinian Authority, listed as “Palestine,” was placed under the category of “Other Organizations” in the official diplomatic roster of the Rome Conference.4

The Palestinian Authority’s General Delegate to Italy, Mr. Nimer Hammad, and the Counselor of the Permanent Observer Mission of Palestine at the United Nations in New York, Mr. Marwan Jilani, did not attend the Rome Conference as representatives of a State, according to the official records of the Conference, but rather under the aegis of “Other Organizations”.5

In accordance with international law and settled diplomatic usage, the Palestinian Authority also did not attempt to vote on the adoption of the final treaty text that established the International Criminal Court at the Rome Conference, as Palestine was not considered a State. So, too, throughout the ICC Preparatory Commission that convened after the adoption of the Rome Statute, the Palestinian Authority again neither attended nor participated as a “country” or “state”, but rather was accepted only under the category of “Entities, intergovernmental organizations and other bodies having received a standing invitation to participate as observers in the sessions and the work of the General Assembly.”6

The same practice was observed, for evident reasons, during the meetings of the ICC Assembly of States Parties over the last seven years, ever since the ICC Statute entered into force on 1 July 2002. At these meetings, the same diplomatic and legal distinctions have been preserved. For example, at the most recent meeting, held at the United Nations Headquarters in New York City from 9th to 13th of February 2009, neither the Palestinian Authority nor “Palestine” was seated in any of the categories applicable to States. They were neither listed as a “State Party to the Rome Statute”, nor as an “Observer State”, nor even as a “State invited to be present”. Rather, the Palestinian Authority or Palestine attended under a fourth category entitled “Entities, intergovernmental organizations, and other entities”.7

Quite apart from the plain meaning of the ICC Statute and the standing diplomatic practice and credential decisions of the Rome conference and the succeeding implementing conferences, there is also the scholarly literature on the ICC Statute that compels this good faith and ordinary reading of the word “State” in Article 12(3).8 As made evident by Rule 44(1) of the ICC Rules of Procedure and Evidence,9 a declaration under Article 12(3) of the ICC Statute can be submitted only by “States” that are otherwise eligible to become Parties to the ICC Statute.

In Article 125 to the ICC Statute, it is noted that only a “State” is eligible for “Signature, ratification, acceptance, approval or accession” to the ICC treaty. Declarations under Article 12 also speak of “acceptance” of the jurisdiction of the Court, and in particular, Article 12(3) invites the retrospective “acceptance” of jurisdiction by a non-party State.10 But the precondition is that it has to be a “State” within the meaning of the treaty and general international law.

As noted in the respected Commentary on the Rome Statute edited by Professor Otto Triffterer, following the Rome conference, “[i]n accordance with normal modern practice for multilateral treaties, the [ICC] Statute [was] open for signature by all States”.11 The commentary notes that not every “entity” qualifies as a “State”, and thus that the range of possible signatories was limited.12

When the period for State signature expired on December 31, 2000, the ICC Statute still remained open to “accession”, but again, only by States. As the Triffterer Commentary notes, in a second edition of this notable treatise, published in 2008,

“States that did not find it possible to sign within the relevant time period may always ‘accede’ to the Statute. Timor-Leste, for example, which had not achieved its independence at the relevant time, was among those acceding to the Statute.”13

But as Professor Triffterer’s treatise indicates, the class of possible members is still strictly limited to States, and does not include Palestine:

A number of small States and Switzerland have joined the United Nations since 1998. In addition to the 191 Members of the United Nations and the Holy See, the lone State that now has Observer Status with the organization, the other entities generally regarded by the international community as States are both small island countries in the Pacific: Cook Island and Niue. … The total potential parties to the Rome Statute thus stands at 193, given the present political status of the World.”14

Professor Triffterer’s treatise excludes the Palestinian Authority and “Palestine”. So, too, the ICC Commentary by Professor Antonio Cassese notes that accession to the treaty, including under Article 12(3), is limited to States.15

Thus, both the wording of the ICC basic documents, the diplomatic practice of the ICC Rome Conference, Preparatory Commission, and Assembly of States Parties, and scholarly commentary make it clear that the Palestinian Authority is not a “State” for the purposes of Article 12(3) of the ICC Statute. This answers conclusively the question raised by submission of the declaration of the Palestinian Authority under Article 12(3).

While no additional general question of international public law arises in this regard, it may nevertheless be worth noting that Article 12(3) is entirely consistent with the practice under other global multilateral treaties and public international law. For example, at the United Nations General Assembly, the representatives of the Palestinian Authority are not permitted to vote. Indeed, unlike the Holy See, Palestine is not included or seated in the category of “Non-member State having received a standing invitation to participate as observer in the sessions and the work of the General Assembly and maintaining permanent observer mission at Headquarters.”16

Rather, in the Official Protocol List of the United Nations, Palestine is listed as belonging to the category of “Entities having received a standing invitation to participate as observers in the sessions and the work of the General Assembly and maintaining permanent observer missions at Headquarters.”17

In addition, general principles of international law concerning the recognition of statehood argue strongly against any different conclusion. For example, the terms of Article 1 of the Montevideo Convention18 establish prerequisites to statehood that include “(a) a permanent population; (b) a defined territory; (c) government; and (d) capacity to enter into relations with the other states.” These criteria are principal indicia of statehood. Yet under the terms of the 1995 interim agreement, one or more of these elements is problematic, since the Palestinian Authority has, for example, foresworn a general capacity to enter into foreign relations with other states.19 So, too, the Palestinian Authority does not effectively control the territory of Gaza.

Furthermore, since the end of the Cold War, there has been a suggestion by some scholars and diplomats that an added normative element should be prerequisite to the recognition of new states. In the early 1990’s, the European Community built upon the Charter of Paris and the Helsinki Final Act, and announced guidelines to regulate the recognition of new states during the break-up of the former Soviet Union and the Federal Republic of Yugoslavia.20

These guidelines suggested that before a new state should be recognized, its record of performance must demonstrate respect for the rule of law, democracy, human rights, and the rights of ethnic minorities, as well as “respect for the inviolability of all frontiers which can only be changed by peaceful means and common agreements.”

Under these standards as well, it would put the cart before the horse to effectively grant statehood before the elements of this test are satisfied. Indeed, it has been noted that premature recognition of a contested territory as an independent state can diminish the incentives for establishing the legal bulwark of these norms.

As a matter of law, the Palestinian Authority was established as a provisional body with clearly defined and circumscribed powers, under a series of agreements between Israel and the Palestinian Liberation Organization, pending the conclusion of permanent status negotiations. Under these agreements, as noted above, the PA lacks capacity in such essential spheres as the conduct of foreign relations.

For example, by the 1995 Israeli-Palestinian Interim Agreement on the West Bank and the Gaza Strip, signed by Israel and the Palestinian Liberation Organization,21 the newly created Palestinian Council is specifically limited in its powers by Article 9(5). In particular, under Article 9(5)(a), the Palestinian Council does “not have powers and responsibilities in the sphere of foreign relations … and the exercise of diplomatic functions.”22

In addition, Article 9(5)(c) of the 1995 Interim Agreement specifically states that “Dealings between the Council [of the Palestinian Interim Self-Government Authority] and representatives of foreign states and international organizations … shall not be considered foreign relations.”

The difference between an agreement to allow substantial internal autonomy and achieving international status as a State has been emphasized by Professor James Crawford, who drafted the original proposal of the ICC Statute while serving as a member of the International Law Commission in 1994. In regard to the status of the Palestinian Interim Self-Government Authority, Professor Crawford notes:

“the limited character of [the Israeli] withdrawal can be seen, for example, from the jurisdiction of the Palestinian Interim Self-Government Authority, which covers: ‘West Bank and Gaza Strip territory, except for issues that will be negotiated in the permanent status negotiations: Jerusalem, settlements, military locations, and Israelis.’ Thus a jurisdiction defined apparently in normal territorial terms is for practical purposes redefined as a jurisdiction over Palestinians (and visitors).”23

Professor Crawford also notes that the commitment of the Palestinian Interim Self-Government Authority to forego any general claim of competence in foreign relations further serves to thwart any plausible claim of statehood.24

While, as Professor Crawford notes, the Palestinian Liberation Organization has served as a representative of the Palestinian people in various international negotiations, the PLO is also not a state.

“The PLO of course is a national liberation organization, widely recognized as such, and is the external representative of the Palestinian people. …There is thus a non-State legal entity recognized as represented by a national liberation movement.”25

Thus, there is no State entity capable of joining the International Criminal Court. As Dr. Crawford notes bluntly, there is an ongoing diplomatic process, but the recognition of a new state of Palestine:

“has never commanded anything like the level of quasi-unanimous support that would be required to establish a particular rule of international law to the effect that Palestine is a State.”26

The international legal formula for creating a future Palestinian State is defined in historic bilateral agreements between Israel and the PLO, which received the active support of the international community. The international community itself (including the United States, the European Union, Russia and the United Nations) has supported the “Roadmap” announced in June 2002 — formally known as the “Performance-Based Roadmap to a Permanent Two-State Solution to the Israeli-Palestinian Conflict”.27

As steps towards reaching that goal, the Roadmap requires steps to be taken by the two sides, including dismantlement of the infrastructure of terrorism on the Palestinian side, institution and capacity-building of Palestinian institutions, and a legitimate government capable of governing the whole Palestinian territory. The Roadmap of course, makes performance demands on Israel as well. This staged process was further reinforced by the international community in the Annapolis conference, as well as by many recent statements of world leaders.

Quite apart from the limiting language of the ICC Statute, the diplomatic practice of the ICC negotiations, and the opinions of scholarly commentators — which do not permit acceptance of the declaration of the Palestinian Authority because it is not a State — it is not the role of the ICC to involve itself in political issues or to truncate that international and bilateral process through a unilateral ascription of statehood, whether direct or implied, countering delicate agreements and on-going international efforts in this matter.

In addition to all of these controlling authorities, there is a central principle of interpretation that argues against any “expansionist” or “teleological” reading of Article 12(3) in this matter. This is the rule of “lenity” in criminal cases, in which the law is not to be read beyond its established meaning.

It may be worth noting the high-level task force report that was issued in March 2009 by the American Society of International Law, following meetings with officials of the International Criminal Court and other figures who were central in the conclusion of the ICC Statute. The task force report properly salutes the Court for its important role in strengthening respect for the principles of humanitarian law, and urges that the United States strengthen its practical support for the work of the Court.28

At the same time, the ASIL task force report also sounds a tocsin warning that the ICC should not stray beyond the established limits of international law, in particular, in regard to jurisdictional decisions.

Notably, the task force remarks that there is a “test for the ICC” in how it handles the declaration submitted by the Palestinian Authority and the claim of accession as a “State” under Article 12(3) of the ICC Statute.29 In the considered view of this group of eminent jurists and former prosecutors, the declaration submitted by the Palestinian Authority

“raises issues about the authority of the Prosecutor, and of the ICC, to treat as a State an entity which is not generally recognized as a State and which is not a U.N. member.30

This decision, suggests the ASIL task force, will necessarily be an important benchmark in evaluating the performance of the Court.

It would be most unfortunate if a general perception of politicization of the Court’s handling of the Article 12(3) declaration by the Palestinian Authority were to take hold. The Court should abide by the established rules of international law, in which the recognition of new states is not a matter delegated to courts. Otherwise, even a preliminary examination by the Office of the Prosecutor may end up raising more serious questions and obstacles in the diplomatic world of real negotiations outside the Court, and raise serious questions for other states on the role and practice of the International Criminal Court. Based on the negotiating history, it is clear that the State Parties that created the Statute did not intend this effect.

Both the Palestinian Authority and Israel have the obligation to investigate any serious violations of humanitarian law and human rights law that have occurred, and in appropriate cases bring to trial those who committed such violations. In the wake of the Gaza military campaign, Israel has stated that it will conduct a national investigation of any disputed actions, to include reviews both by military and civilian authorities, with further review available in the High Court of Justice in Israel. Likewise, the Palestinian Authority has the responsibility, and will wish to investigate and bring charges against any persons or organizations in Gaza that have launched thousands of rockets against the territory of Israel over the course of a multi-year terror campaign, and allegedly used Gaza citizens as human shields.

The solution of the Israeli-Palestinian conflict is something that all people of good faith should devoutly seek. But the jurisdictional basis of a criminal tribunal must also be founded on the steadiness of the law. Neither the Palestinian Authority nor the “entity” of Palestine currently qualifies as a State within the meaning of the Rome Statute. Hence, the declaration submitted on January 22, 2009 cannot be accepted by the Court.

Endnotes — (click the footnote reference number, or ↩ symbol, to return to location in text).

  1. 1.

    See “Declaration recognizing the Jurisdiction of the International Criminal Court” (21 January 2009) from Mr. Ali Khashan, Minister of Justice, Palestinian National Authority, to the ICC Registrar, available online. The Registrar of the Court received this declaration on 22 January 2009.

  2. 2.

    In its first full paragraph, the Declaration reads as follows: “In conformity with Article 12, paragraph 3, of the Statute of the International Criminal Court, the Government of Palestine hereby recognizes the jurisdiction of the Court for the purpose of identifying, prosecuting and judging the authors and accomplices of acts committed on the territory of Palestine since 1 July 2002.” It further states that “This declaration, made for an indeterminate duration, will enter into force upon its signature.”

  3. 3.

    The ICC Statute, the records of the Rome Conference, the ICC Rules of Procedure and Evidence and the Regulations of the Prosecutor’s office are equally clear. See the ICC Rules of Procedure and Evidence, Official Records ICC-ASP/1/3, Rule 44; and the Regulations of the Office of the Prosecutor, ICC-BD/05-01-09, 23 April 2009, Regulation 25(1)(c).

  4. 4.

    See the Official Records of the United Nations Diplomatic Conference of Plenipotentiaries on the Establishment of an International Criminal Court, A/CONF.183/13, volume II, pp. 5 and 44.

  5. 5.

    See id. at 44.

  6. 6.

    See, e.g., Preparatory Commission for the International Criminal Court, 9th session, New York, 8–19 April 2002, PCNICC/2002/INF/6, 30 April 2002 (emphasis added).

  7. 7.

    See International Criminal Court, Assembly of States Parties, 7th session (second resumption), New York, 9–13 February 2009, ICC-ASP/7/INF.1/Add.2. Compare Vienna Convention on the Law of Treaties, Mary 23, 1969, 1155 U.N.T.S. 331, Article 31(3)(b).

  8. 8.

    Vienna Convention on the Law of Treaties, supra note 7, Article 31(1).

  9. 9.

    See Article 44(1) of the ICC Rules of Procedure and Evidence: “The Registrar, as the request of the Prosecutor, may inquire of a State that is not a Party to the Statute or that has become a Party to the Statute after its entry into force, on a confidential basis, whether it intends to make the declaration provided for in article 12, paragraph 3” (emphasis added).

  10. 10.

    The declaration filed by the Palestinian Authority fails to track the exact language of Article 12(3), rather speaking of its “recognition” of the Court’s jurisdiction for an “indeterminate duration” rather than its “acceptance”— but it does reference Article 12(3).

  11. 11.

    Otto Triffterer, editor, “Commentary on the Rome Statute of the International Criminal Court, First Edition,” at p. 1287 (Nomos Verlagsgesellschaft, Baden-Baden, Germany, 1999 edition) (emphasis added).

  12. 12.

    Id. at 1287 note 1.

  13. 13.

    Otto Triffterer, “Commentary on the Rome Statute of the International Criminal Court, Second Edition,” at p. 1773, (Hart Publishing Company, Oxford, 2008 edition) On April 8, 2009, Professor Triffterer won the Certificate of Merit of the American Society of International Law “for high technical craftsmanship and utility to practicing lawyers and scholars” for this volume.

  14. 14.

    Id. at 1773 note 1 (second edition).

  15. 15.

    Antonio Cassese, Paola Gaeta, John R.W. D. Jones, “The Rome Statute of the International Criminal Court: A Commentary” (Oxford University Press 2002), vol. 1, p. 1203.

  16. 16.

    See, e.g., Executive Office of the Secretary-General, Protocol and Liaison Service, Publication of Permanent Missions to the United Nations, No. 297, July 2007, at p. 293.

  17. 17.

    See, e.g., Executive Office of the Secretary-General, Protocol and Liaison Service, Publication of Permanent Missions to the United Nations, No. 297, July 2007, at p. 294.

  18. 18.

    See Convention on the Rights and Duties of States (Montevideo Convention), December 26, 1933, 165 League of Nations Treaty Series (LNTS) 19; 28 American Journal of International Law (Supplement) 53 (1934).

  19. 19.

    See “Israeli-Palestine Liberation Organization: Interim Agreement on the West Bank and the Gaza Strip,” Sept. 28, 1995, article 9(5), 36 International Legal Materials 551, 561 (1997) [hereinafter cited as Interim Agreement].

  20. 20.

    See “Guidelines on the Recognition of New States in Eastern Europe and the Soviet Union”, Extraordinary EPC Ministerial Meeting, Brussels, 16 December 1991, EC Bulletin, 12-1992, p. 199; UN Doc. S/2393 of 17 December 1991, Annex II; International Legal Materials 31 (1992), p. 1486.

  21. 21.

    See Interim Agreement, supra note 19.

  22. 22.

    Id. at art. 9(5)(a). The only exception is for “economic agreements,” “agreements with donor countries”, “agreements for the purpose of implementing … regional development plans”, and “cultural, scientific and educational agreements”. See Article 9(5)(b). This is comparable to the ability of entities and international organizations, which are not States, to enter into such agreements.

  23. 23.

    James Crawford, “The Creation of States in International Law” (Oxford University Press, second edition, 2006, paperback edition 2007), at pp. 443–4 (citing the Agreed Minutes to the Declaration of Principles on Interim Self-Government Arrangements, B, Art. IV: (1993), 32 International Legal Materials 1542, “repeated with further elaboration in Art XVII … of the Interim Agreement on the West Bank and the Gaza Strip, 28 September 1995, which states that “[i]n accordance with the Declaration of Principles, the jurisdiction of the Council will cover West Bank and Gaza Strip territory as a single territorial unit, except for: (a) issues that will be negotiated in the permanent status negotiations: Jerusalem, settlements, specified military locations, Palestinian refugees, borders, foreign relations and Israelis.”).

  24. 24.

    Id. at 444.

  25. 25.

    Id. at 444 (emphasis added).

  26. 26.

    Id. at 438 (emphasis added).

  27. 27.

    Reproduced online.

  28. 28.

    The ASIL Task Force issued its report, entitled “U.S. Policy Toward the International Criminal Court: Furthering Positive Engagement,” on 21 March 2009. The co-chairs of the task force were Patricia Wald, former chief judge of the U.S. Court of Appeals for the District of Columbia Circuit and former judge of the International Criminal Tribunal for the former Yugoslavia, and William H. Taft IV, former legal adviser of the U.S. Department of State. Task force members included, inter alia, retired U.S. Supreme Court Justice Sandra Day O’Connor, former president of the International Court of Justice Stephen Schwebel, and David Tolbert, former deputy prosecutor of the International Criminal Tribunal for the former Yugoslavia.

  29. 29.

    Id. at pp. vi-vii.

  30. 30.

    Id. (emphasis added).

  31. Suggested Citation for this Comment:

    George P. Fletcher, No Jurisdictional Basis for an Investigation Pursuant to the Palestinian Declaration, ICC Forum (Sep. 1, 2010), available at https://iccforum.com/gaza#Fletcher.

    Suggested Citation for this Issue Generally:

    Should the ICC Investigate War Crimes in Gaza?, ICC Forum (Sep. 1, 2010), available at https://iccforum.com/gaza.

Dr. Marlies Glasius Avatar Image Dr. Marlies Glasius Senior Lecturer University of Amsterdam

Press Releases, Not Arrest Warrants: Interpreting the ICC Prosecutor’s Moves in Relation to the Gaza Situation

The actual prosecution and trial of perpetrators of war crimes, crimes against humanity or genocide is only one aspect of the ICC’s power in the world.

Summary

Unlike the other contributions to this Forum, this article will not attempt to assess whether or not the prosecutor of the International Criminal Court (ICC) has legal authority over war crimes committed in the Gaza war. Instead, it will make a socio-political contribution to the debate, in three ways. First, it will consider in detail what the prosecutor of the ICC, Luis Moreno-Ocampo, has done about the Gaza file since January 2010, interpreting his actions from the perspective of symbolic politics rather than international law.

Second, it will consider, by comparing the Gaza situation to the ICC’s other ‘files’, whether these actions may be having positive socio-political effects on the ground in Gaza and Israel. A disclaimer must be made straight away at this point: this article is based on extrapolation from and comparison with research done by myself and others in the Central African Republic, Colombia, Democratic Republic of Congo and Uganda, not on research done in Israel or Gaza. Its point of departure will be that war crimes were committed on both sides, but that the number of victims on the Palestinian side has been very much higher.1

Third, it will consider the motivations behind, and constraints on, the prosecutor’s actions, and discuss his future options in relation to the Gaza situation.

Argument

The Prosecutor’s actions to date

In January 2009, confronted with the Palestinian National Authority’s declaration to the effect that it recognized the jurisdiction of the ICC, the Registry of the ICC acknowledged receipt in non-committal terms: ‘without prejudice to a judicial determination on the applicability of article 12, paragraph 3 to your correspondence, I wish to inform you that a declaration under article 12, paragraph 3 has the effect of acceptance of jurisdiction’.2

On this occasion, the ICC prosecutor met with Palestinian justice minister Ali Khashan, and two weeks later again with Khashan as well as with foreign minister Riad al-Malki. On both occasions, the Prosecutor sent out press releases with photographs of himself shaking hands or holding documents with the Palestinian officials. The second press release stated that he had now received 326 communications relating to the Israel/Palestine situation since 27 December 2008, and both promised that his office would ‘carefully examine’ all relevant issues relating to jurisdiction.3

In July 2009, after the presentation of the Dugard Report,4 the prosecutor visited Arab League headquarters to discuss the matter.5

In October 2009, after the Goldstone Committee recommended that the UN Security Council refer crimes committed by both sides in the Gaza war to the ICC, the Prosecutor met again with Palestinian officials and members of the Arab League, as well as with the Dugard commission and with other international lawyers.6

More significantly, he was reported in Newsweek to have stated that he had authority to prosecute David Benjamin, an officer with dual Israeli and South African citizenship, who served in the Military Advocate General’s international law department, which authorized which targets troops would strike before and during the Gaza war.7 Benjamin had visited South Africa in August, sparking a campaign for his arrest and causing him to return to Israel.8 No press release was issued confirming the statement made to Newsweek, and no further action, at least no public action, has been taken in relation to David Benjamin since.

From a legal perspective, the Prosecutor has done nothing very significant in relation to crimes committed in the 2008–2009 Gaza war.

In January 2010, the Prosecutor published on the ICC website his response to an inquiry by the United Nations High Commissioner for Human Rights, stating again that it was analyzing whether the ICC could have jurisdiction, summarizing the information his office had received regarding jurisdiction as well as substantive evidence, and pointing out that there is no deadline in the Rome Statute for making a determination.9

In May 2010, he published a summary of submissions made on the issue of jurisdiction, annexed by a list of most significant submissions. The document is what its title describes, a summary, relating what some submissions say, and what others say, but nowhere referring to any possible conclusions on the side of the ICC.10

In other words, from a legal perspective, the Prosecutor has done nothing very significant in relation to crimes committed in the 2008–2009 Gaza war. Yet he has periodically raised media attention, associating crimes committed in the Gaza war with the idea of ICC prosecution, and has even named a ‘virtual suspect’, whilst ‘analyzing’ the jurisdictional issues involved for over eighteen months. Why?

Potential sociopolitical effects

I have argued elsewhere11 that the actual prosecution and trial of perpetrators of war crimes, crimes against humanity or genocide is only one aspect of the ICC’s power in the world, and that it may in fact be the most disappointing and least important aspect.

The ICC will never be able to try more than a handful of suspects, and who these are will necessarily be a function of who is subject to the Court’s jurisdiction, and who can actually be apprehended, as much as who in the world is most guilty.

In the seven years that the ICC has been up and running, it has served 13 arrest warrants (two Ugandan suspects have since died), received three volunteer suspects, and only actually arrested four people, all Congolese warlords. The first, nearly aborted, trial on the narrow charge of using child soldiers has yet to see a verdict. From the perspective of retributive justice, the record of the ICC to date looks rather dismal. This will get better, as there will eventually be more trials and presumably also some convictions, but it will not get very much better. The ICC will never be able to try more than a handful of suspects, and who these are will necessarily be a function of who is subject to the Court’s jurisdiction, and who can actually be apprehended, as much as who in the world is most guilty.

But that is not to say that the five investigations to date, and the threat of investigations elsewhere, have been without positive, justice-enhancing socio-political consequences. In relation to these other cases, the mere existence of the ICC, and the announcement that the prosecutor is considering opening an investigation, can have the following positive effects:

  • drawing attention to, and raising the profile of, human rights violations in a particular situation; mobilizing victims and raising their self-esteem
  • opening public debates about guilt and accountability that might otherwise have remained closed; in particular constructing responsibility for crimes in terms of individual rather than collective guilt
  • triggering more serious national investigations and prosecutions to prevent ICC involvement

I have argued that these ‘unintended consequences’ of the ICC have been and will continue to be far greater than its legal authority and capacity would suggest.

I have argued that these ‘unintended consequences’ of the ICC have been and will continue to be far greater than its legal authority and capacity would suggest.12 I will now attempt assess whether any such positive effects may be expected from the mere association between crimes committed in the Gaza war and the prospect of ICC prosecution, raised by the prosecutor’s subtle actions to date. This assessment comes with an immediate disclaimer: the author is by no means an expert on the situation in Gaza or Israel, and is merely extrapolating and speculating on whether effects seen in other situations may apply in this case as well.

Catalyst for world attention and victims’ sense of agency?

The first effect, that of drawing attention to, and raising the profile of, human rights violations in a particular situation, has been of great value in the Central African Republic, and perhaps to a lesser extent in Darfur, DRC and Uganda.13 In the case of the Gaza war, this effect is highly doubtful. There may be a lot of controversy surrounding crimes committed in the Gaza war, but they have not escaped notice. Practically every state, international organization and numerous civil society organizations have responded both to the initial crimes and to the subsequent reports of the two independent fact-finding commissions. The people of Gaza did not need the ICC merely to draw attention to what had happened.

The second effect, that of mobilizing victims and raising their self-esteem has again been particularly evident in the case of the CAR, in a context where many of the crimes committed were of a sexual nature.14 Different assessments could be made about how mobilized, and in what ways, the people of Gaza are, and how their self-esteem may be affected by their hopeless situation. It is clear, however, that the Palestinians in the Occupied Territories have been the subject of decades of solidarity campaigns in the Arab world, in the developing world and in the West.

The Goldstone commission has already given official UN validation to the crimes against them, as well as the lesser crimes against Israeli citizens, in the Gaza war. Amidst decades of demonstrations, boycotts, letters and legal action, including some by Israeli activists, the unlikely prospect of ICC prosecutions is not a unique mobiliser. Even the opening of an investigation might not be distinguishable, by the average Palestinian, from the numerous unsuccessful attempts at pursuing universal jurisdiction suits against Israeli officials in various national courts. Only actual arrests might be substantially different.

Catalyst for accountability debates?

The second effect, that of opening debate, would at first sight seem one that the Gaza situation is little in need of. The actions of the Israeli army against Palestinians, and the actions of Hamas against Israeli civilians, have been the subject of extensive, virulent debate, in Israel and elsewhere. By and large, however, these debates can hardly be credited with being informed, civil, or constructive.

I have argued, in the wake of the Gaza war, that the intimation of an ICC investigation might bend the debate in Israel itself in a particular, more constructive direction, suggesting that the allocation of individual rather than state responsibility might widen the space for debating the morality and legality of the Gaza bombings in Israel itself, because it is less immediately associated with debates on Israel’s right to exist and to defend itself. Collective attribution of responsibility, I argued, is open to being read as implying that a) every Israeli citizen (including for instance Arab Israelis or conscientious objectors) shares in the guilt and b) that Israel, being guilty, has no right to exist. This collectivization serves to strengthen siege mentality and self-justification, and shrinks the space for critical voices. The ICC on the other hand could disaggregate exactly which political or military leaders could be considered ‘the most responsible’ in relation to specific crimes.15

In the former Yugoslav republics, although initial local reactions to the Yugoslavia Tribunal were intensely hostile, the long-term effect of indictments has precisely been to marginalize some of the most hawkish individuals and shrink the space for denial of what exactly happened in the recent wars.16 The focus on individual responsibility might therefore also be more promising in helping Israeli citizens reflect and speak out loud about whether the Gaza attacks occurred, and similar attacks should continue to occur ‘in their name’.

However, eighteen months later I must conclude that this effect has not occurred to date. There is little reason to believe that most Israeli’s are carefully distinguishing between accusations against the state of Israel and against individual soldiers and politicians. Insofar as there has been any public debate on possible ICC involvement, one finds rather the opposite. Coverage by the (liberal) Haaretz newspaper of the Newsweek report about Lt. Col. Benjamin for instance provoked the online comment “We are all David Benjamin”, enthusiastically endorsed by subsequent posts.17

Catalyst for internal investigations?

The prosecutor of the ICC famously said when he began his term that the success of the ICC should be measured not by the number of prosecutions, but by their absence.18 What he had in mind was not so much outright prevention as domestic prosecution of crimes. This ‘positive complementarity’ effect is supposed to occur both in situations where an investigation has been opened and in situations that are under consideration (and perhaps even in situations where there is no such threat). The latter appears to be the strategy in relation to Colombia as well as Georgia. In both cases, there are ongoing proceedings which could with some latitude be interpreted as meeting the ‘genuine willingness’ criteria that would render an ICC case inadmissible.19 Further research would be required to establish whether there is indeed a causal relation between the prosecutor’s actions and the vigour with which these proceedings are pursued, but a credible argument can certainly be made that such an effect could occur in certain cases.

In relation to the Gaza war, there has certainly not been an absence of domestic investigations. In its response to the Goldstone Report, the Israeli government make mention of “more than 100 incidents…15 Military Police criminal investigations…eight additional criminal investigations into forty incidents…and twenty three additional Military Police criminal investigations.”20 At that point, there had however been only one conviction, for theft. There is no doubt then about the quantity of investigations. Their quality is the subject of virulent contestation amongst lawyers and human rights activists. The question for the purpose of this article is, however, a different one: did the prosecutor of the ICC have anything to do with this flurry of investigations?

There is little reason to believe this to be the case. First of all, the large number of investigations following the Gaza war is not unique. According to the Israeli government’s own information, 1,467 criminal investigations were opened into alleged misconduct by soldiers, leading to the issuance of 140 indictments against soldiers regarding crimes committed against the Palestinian population, between 2002 and 2008.21 Secondly, Israeli officials devote a great deal of detailed attention to disputing the results of the Goldstone commission, whilst the ICC receives little mention, either in official documents or in the press. Finally, these official defences of operation Cast Lead are primarily framed in terms of the state of Israel’s conformity with international law.

Indeed, whilst the prosecutor has requested information from both sides about ongoing national proceedings, he has, unlike in the Colombian, Georgian or Kenyan case, refrained from remarks to the effect that there have to be such proceedings, nor has he attempted to take credit for, or commented on the quality of, the Israeli investigations.

Then what is the point? The ICC’s quest for legitimacy and relevance

Eighteen months after the Gaza war, there is no indication that the prosecutor of the ICC will actually seek permission from the pre-trial chamber to open an investigation in relation to the Gaza war. The premise of this article has been to investigate whether the mere hints at investigation thrown out by the prosecutor appear to have had any of the positive socio-political effects on the situation that have been observed in other situations.

The answer is a resounding ‘no’. The human rights violations that took place in the Gaza war were not a forgotten incident that needed to be brought to the attention of the world by the ICC. Its victims were not galvanized by the (unlikely) prospect of seeing justice done in the ICC. The prosecutor’s cautious expression of interest in the situation did not substantially influence the public debate in Israel. There is no reason to believe that it had a preventive effect on future abuse, and while there are ongoing national proceedings, these cannot be interpreted as having been prompted by the interest of the prosecutor of the ICC. Then why has the prosecutor taken the situation into consideration, and taken action every few months to renew attention to the fact that he is still analyzing it? I would argue that while Gaza does not need the ICC, the ICC, and more particularly its prosecutor, needs Gaza.

I would argue that while Gaza does not need the ICC, the ICC, and more particularly its prosecutor, needs Gaza.

One of the most frequent questions non-experts in Europe and elsewhere ask about the ICC, is why no Americans have been prosecuted for the invasion of Iraq, or for suspected crimes committed in the course of that war. It is a question that is of course easy to answer in terms of jurisdiction determined by the Rome Statute, but the next question, what the point is of a Court that can investigate some situations but not others, and more particularly not the actors perceived as most powerful in world politics, is not so easy to answer. The jurisdictional constraints of the Court impinge on its perceived legitimacy with various publics. More particularly, the ICC has in recent years been much criticized for its apparently exclusive focus on Africa, by academics, civil society groups and African states. The fact that the United States remains a non-party, and continues to view the ICC with moderate hostility, does little to diminish the perception of a hegemonic, pro-Western court whose purpose is to discipline African rebels and rogue states.

It is precisely with these audiences that an investigation of the Gaza war (and particularly of course, of crimes committed by the Israeli side) would play extremely well, and could do much to redeem the reputation of the ICC as a universal instrument of justice. Indeed, even states that have not ratified the ICC Statute and never been sympathetic to it may now include a reference to the ICC in their ritual anti-Israel policies.22

Quo Vadis, Prosecutor?

However, as the other contributions to this forum will undoubtedly be pointing out, seeking the pretrial chamber’s approval to open an investigation on a territorial basis would be highly controversial in legal terms. In political terms, it would alienate the United States, recently more cooperative than under the Bush administration, as much as it would gratify pro-Palestinian audiences and non-western states.

This dilemma explains why the prosecutor has been stringing out the ‘analysis’ of whether and how the ICC can exercise territorial jurisdiction in the Gaza situation. Complex though the issue may be, as an intellectual exercise surely it should be within the capabilities of the prosecutor’s legal staff to answer the question within a year. Instead, the letter sent to the UN High Commissioner for Human Rights last January pointed out that the ICC Statute does not specify any deadline for a decision on whether to open an investigation, implying that the ‘analysis’ might be prolonged indefinitely.23

Any legitimacy gains the ICC may have made with certain audiences by making a show of considering the situation cannot be sustained indefinitely.

However, while perpetual analysis might be the safest course of action in terms of avoiding controversy, any legitimacy gains the ICC may have made with certain audiences by making a show of considering the situation cannot be sustained indefinitely. Eventually, the refusal to make a determination either way will be interpreted as cowardly. If we liken the prosecutor’s actions in relation to the Gaza situation to a game of chess, we must ask, how many moves has he thought ahead?

One alternative route to prosecution, which he has already pointed to, would be the identification of suspects with dual citizenship with an ICC-ratifying state, such as South Africa. If accepted by the pre-trial chamber, this would allow for an arrest warrant to emerge from the Gaza war whilst sidestepping the question of Palestinian statehood. This would probably still mar the rapprochement with the US, but not as much as the territorial route.

However, such course of action raises its own problems. First, it would deviate from the prosecutor’s own policy of pursuing ‘the most responsible’, replacing it with a morally arbitrary policy of pursuing ‘those who happen to have dual nationality’. The apprehension of minor suspects on this criterion would be unlikely to instill a sense of justice being served among Palestinians, let alone Israeli’s. Second, unless the prosecutor gets lucky and finds a suspect of dual nationality recklessly hanging around in a cooperative state, it is unlikely that such an arrest warrant could be served. In effect, like the arrest warrant against President Al-Bashir of Sudan, it would function as little more than a travel ban. It is unlikely that a warrant that cannot be served would unleash any of the positive socio-political effects discussed above. Moreover, adding yet again to the number of suspects whose arrest warrant is a dead letter will make the ICC look less, rather than more relevant and legitimate as an actor in world politics.

The prosecutor’s attempt to establish a mental association between his office and Palestinian victims in the wake of the Gaza war may at the time have appeared like a clever public relations strategy serving to obscure and mitigate the ICC’s legal and realpolitical limitations. But many months of inaction later, it is actually making these limits all the more visible.

Endnotes — (click the footnote reference number, or ↩ symbol, to return to location in text).

  1. 1.

    In this I follow both the Dugard Report and the Goldstone Report: Report of the Independent Fact Finding Committee on Gaza: No Safe Place, Presented to the League of Arab States, 30 April 2009, p.3, available online, last visited 28 July 2010; Report of the United Nations Fact-Finding Mission on the Gaza Conflict, UN Doc. A/HRC/12/48, 25 September 2009, p.17, available online, last visited 28 July 2010.

  2. 2.

    Letter, Silvana Arbia, Registrar, International Criminal Court, to Palestinian National Authority, 23 January 2009, available online, last visited 28 July 2010.

  3. 3.

    Press Release, “OTP ICC, Visit of the Minister of Justice of the Palestinian National Authority, Mr Ali Khashan,” 22 January 2009, available online, last visited 28 July 2010;

    Press Release, “OTP ICC, Visit of the Palestinian National Authority Minister of Foreign Affairs, Mr. Riad al-Malki, and Minister of Justice, Mr. Ali Khashan, to the Prosecutor of the ICC,” 13 February 2009, available online, last visited 28 July 2010.

  4. 4.

    Report of the Independent Fact Finding Committee, cf. note 1.

  5. 5.

    Letter, Béatrice Le Fraper du Hellen, Director, Jurisdiction, Complementarity and Cooperation Division, OTP ICC, to Kyung Wha Kang, Deputy High Commissioner for Human Rights, 12 January 2010, available online, last visited 28 July 2010.

  6. 6.

    Press Release, “OTP ICC, ICC Prosecutor Receives Palestinian Minister of Justice, Arab League and Independent Fact-Finding Committee,” 16 October 2009, available online, last visited 28 July 2010.

  7. 7.

    Dan Ephron, “ICC Prosecutor May Charge Israeli With War Crimes”, Newsweek 21 September 2009.

  8. 8.

    Yotam Feldman, “ICC May Try IDF Officer in Wake of Goldstone Gaza Report”, Haaretz, 24 September 2009.

  9. 9.

    Letter, Le Fraper du Hellen, cf. note 5.

  10. 10.

    OTP ICC, “Situation in Palestine: Summary of submissions on whether the declaration lodged by the Palestinian National Authority meets statutory requirements,” 3 May 2010, available online, last visited 28 July 2010.

  11. 11.

    Marlies Glasius, “We Ourselves, We Are Part of the Functioning”: The ICC, Victims, and Civil Society in the Central African Republic. African Affairs. Vol.108, No.430, January 2009, 49–67; Marlies Glasius, The ICC and the Gaza war: legal limits, symbolic politics, Open Democracy, 28 March 2009, available online, last visited 28 July 2010; Marlies Glasius, 2009a ‘What is Global Justice and Who Decides?: Civil Society and Victim Responses to the International Criminal Court’s First Investigations’. Human Rights Quarterly. Vol.31, No.2, May 2009, 496–520.

  12. 12.

    On the other hand, I have also pointed out obstacles to these effects, and even negative effects: the aspirations of victims for rehabilitation may be converted into disillusionment if justice is done too little, too slowly, and with too little ownership on their part; legal rehabilitation may be of little value to victims when their material and physical security remains under threat; the potential of debates on accountability may not be realized if the organs of the Court, including the prosecutor, do not foster and contribute to these debates.

  13. 13.

    See Glasius, “We Ourselves” and Glasius, “What is Global Justice?”, note 11.

  14. 14.

    See Glasius, “We Ourselves”, note 11.

  15. 15.

    See Glasius, “The ICC and Gaza”, note 11, for a more extensive version of this argument.

  16. 16.

    Diane F. Orentlicher, “Shrinking the Space for Denial: The Impact of the ICTY in Serbia,” New York: Open Society Institute, May 2008.

  17. 17.

    Online comment 20 et al. to Feldman, note 9, available online, last visited 28 July 2010.

  18. 18.

    ICC OTP, Paper on some policy issues before the Office of the Prosecutor, September 2003, 4, available online, last visited 28 July 2010.

  19. 19.

    Maartje Stabel, “Massacre in Colombia: Ending Impunity with the Justice and Peace Law,” July 2010, Masters’ Thesis on file with the author; Amnesty International, “Civilians in the Aftermath of War: The Georgia-Russia conflict One Year On,” 7 August 2009, 26–27, available online, last visited 29 July 2010.

  20. 20.

    Ministry of Foreign Affairs, Government of Israel, Initial Response to Report of the Fact Finding Mission on Gaza Established Pursuant to Resolution S-9/1 of the Human Rights Council, 24 September 2009, p.21–22, note 23, available online, last visited 29 July 2010.

  21. 21.

    State of Israel, “The Operation In Gaza: Factual And Legal Aspects,” July 2009, 109, available online, last visited 29 July 2010.

  22. 22.

    The Malaysian parliament for instance unanimously adopted a resolution asking Turkey to refer the recent Israeli attack on the aid ship Marmara to the ICC. Melissa Goh, “Malaysia wants Israel referred to ICC over Gaza flotilla raid”, Channel News Asia, 7 June 2010, available online, last visited 21 July 2010. Other news reports suggesting that Turkey does indeed want to refer the flotilla raid to the ICC appear to be confusing it with the International Court of Justice.

  23. 23.

    See note 5.

  24. Suggested Citation for this Comment:

    Marlies Glasius, Press Releases, Not Arrest Warrants: Interpreting the ICC Prosecutor’s Moves in Relation to the Gaza Situation, ICC Forum (Sep. 1, 2010), available at https://iccforum.com/gaza#Glasius.

    Suggested Citation for this Issue Generally:

    Should the ICC Investigate War Crimes in Gaza?, ICC Forum (Sep. 1, 2010), available at https://iccforum.com/gaza.

Michael Kearney Avatar Image Dr. Michael Kearney Lecturer in Law University of Sussex

It is imperative that in considering the issue at hand, the correct question be asked, and that Palestinian jurisdiction over crimes set out in the Rome Statute be acknowledged as giving the ICC the ability to accept the PA declaration.

The crucial element [is to determine] whether Palestine as an entity has the capacity to validly transfer jurisdiction to the Court.

Summary

A striking feature of the Summary of Submissions published by the Prosecutor’s office is the dichotomy in the approaches taken by the various authors. Four of the eight papers apply a rigid and strict interpretation of the 1933 Montevideo Convention to conclude that Palestine is not a state and therefore the declaration must be rejected. The remaining papers conclude that the declaration can and should be accepted as valid by the Prosecutor. To these authors the Montevideo Convention is of limited, if any, significance, with the emphasis placed instead on the Palestinian right to self-determination and their sole sovereignty over the occupied Palestinian territory, as well as widespread implicit and explicit recognition of Palestine either as a state, or as an international legal personality that in relevant ways is the equivalent of a state. The crucial element to this approach is that the question for the Prosecutor and for the Court is not one of determining whether Palestine is a state per se and for every purpose, but rather to determine whether in light of the Rome Statute, Palestine as an entity with the capacity to enter into relations with states and international organizations, and with jurisdiction over the crimes set forth in the Rome Statute, has the capacity to validly transfer that jurisdiction to the Court.

Argument

Palestine and the International Criminal Court: Asking the Right Question

The theme to this volume is whether the Prosecutor of the ICC has the authority to open an investigation into alleged crimes committed in the 2008–2009 Gaza conflict. The basis of the discussion is the Palestinian declaration under article 12(3) of the Rome Statute submitted in January 2009. The significance of the matter to the promotion of accountability, and the relevance of international law more generally – as well as the resolution of the Israeli-Palestinian conflict – is immense.

A striking feature of the Summary of Submissions published by the Prosecutor’s office is the dichotomy in the approaches taken by the various authors. Four of the papers apply a rigid and strict interpretation of the 1933 Montevideo Convention to conclude that Palestine is not a state and therefore the declaration must be rejected since only states can engage with the International Criminal Court. The remaining papers conclude that the declaration can and should be accepted as valid by the Prosecutor. To these authors the Montevideo Convention is of limited, if any, significance, with the emphasis placed instead on the Palestinian right to self-determination and their sole sovereignty over the occupied Palestinian territory, as well as widespread implicit and explicit recognition of Palestine either as a state, or as an international legal personality that in relevant ways is the equivalent of a state. The crucial element to this approach is that the question for the Prosecutor and for the Court is not one of determining whether Palestine is a state per se and for every purpose, but rather to determine whether in light of the Rome Statute, Palestine as an entity with the capacity to enter into relations with states and international organizations, and with jurisdiction over the crimes set forth in the Rome Statute, has the capacity to validly transfer that jurisdiction to the Court.

Summary of the Submissions to the OTP

Prior to the publication of the OTP’s summary of arguments on 6 May 2010 and the accompanying Annex, academic consideration of the matter had been quite limited. A debate at Rutgers Law Record revolved around whether Palestine was in fact a state or not, with John Quigley making a quite convincing argument that Palestine is a state, an argument repeated in his submission to the OTP. Robert Weston Ashe argued to the contrary, relying in large point on his claim that Quigley ‘utterly failed to consider a long series of official statements (including very recent ones) by top Palestinian officials that openly and repeatedly disclaim that Palestine is currently a “State.”1

I do not believe this particular issue to be the deal breaker that others suggest it is,2 yet beyond representing an understandable mis-interpretation of what the Palestinian political leadership is saying, it highlights the difference between the ‘orthodox’ and ‘functional’ approaches that characterize the two schools of thought to be observed in the papers submitted to the Prosecutor. While those following an orthodox approach repeatedly emphasize such statements, they are wholly absent from the submissions of those advocating for acceptance of the declaration.

Malcolm Shaw, in an Opinion commissioned by the International Association of Jewish Lawyers and Jurists, places substantial store by the absence of any formal Palestinian claim to statehood that has been ‘a consistent feature of Palestinian Authority and PLO practice, particularly since 1993, to argue for a Palestinian state to be created’.3 Similarly the European Centre for Law and Justice paper stresses statements such as Palestinian Authority president Mahmoud Abbas’s references to Jerusalem as ‘the capital of our future independent state’.4

A more attentive reading of such statements suggests an alternative interpretation since such statements do not stress a demand for a Palestinian state, but rather for ‘independence’ and ‘viability’. It is incorrect to read into such statements a renunciation of statehood, sine the central message instead is the demand for an end to the occupation that denies independence to the Palestinians.

On 30 May 2010 Saeb Erekat, chief negotiator of the PLO requested in meetings with representatives of eight nations that ‘Countries that have not recognized Palestine and failed to upgrade PLO missions to embassies should do so “instantly, without hesitation”. He further noted that “Israel continues to evade implementing requirements of peace, including bringing an end to the occupation, and establishing an independent Palestinian state with East Jerusalem as its capital.”5 What is to be made of such a seemingly contradictory statement other than that the PLO/PA considers Palestine to be a state and that they wish for increased recognition by other states in order to help force an end to the occupation and thereby facilitate Palestinian independence? Such is the position of the Arab League, to which Palestine was admitted as a member by route of a special procedure to allow it to participate in the League’s work ‘until that country enjoys actual independence’, since ‘[h]er existence and her independence among the nations can, therefore, no more be questioned de jure than the independence of any of the other Arab States’.6

The existence of such a marked contrast concerning the significance or otherwise of these political statements is a marker for the distinct philosophies embodied by the orthodox and functional approaches. Crudely put, the orthodox approach seems satisfied to accept words at face value, without any attempt at contextualisation or reflection and reasoning, whether they be the statements of politicians or the texts of international treaties. The following sections will consider the key elements of both approaches, beginning with the Orthodox approach, and critiquing its mechanical, conservative character, before moving to discuss, and endorse, the practical functional approach to the questions raised by the Palestinian declaration.

An Orthodox Approach

Four of the submissions rely almost entirely upon similar interpretations of the Montevideo Convention criteria of statehood to conclude that Palestine is not a state for the purposes of international law and therefore the Palestinian declaration must be rejected. The IAJLJ paper does consider, and reject, the idea that the meaning of state for the purpose of the Rome Statute may be any different than it is for international law and international relations in every sense.7 Advocates of the Montevideo approach argue that only states can submit valid article 12(3) declarations, a state is an independent entity that satisfies the Montevideo criteria, Palestine is neither independent, nor a member state of the UN. Since it fails to meet the criteria, the declaration must be rejected.

The Montevideo Convention of 1933 claimed that the four prerequisites to statehood are 1) permanent population 2) defined territory 3) government 4) capacity to enter relations with other states.8 While Palestine is generally accepted as meeting the first two criteria, critics stress a failure to satisfy the latter pair, interpreting government strictly to mean effective control over all the territory in question. Pointing to the limitation of Palestinian exercise of control in the West Bank due to its division under the Oslo Accords into Areas A, B, & C, over which the PA exercises varying degrees of control, and more pointedly to the division of political control between Gaza and the West Bank, it is held that Palestine does not satisfy this requirement.9 David Davenport has argued that even if Palestine were a state, failure to exercise effective government over Gaza is adequate to deny the validity of the PA declaration.10 I have considered and dismissed this suggestion elsewhere, having reference for example to the situation of Cyprus but it does need to be highlighted that both the Fatah and Hamas political organizations continue to consider the Palestinian Authority as the legitimate framework and manifestation of the Palestinian polity throughout the oPt.11 Mendes, critical of the orthodoxy whereby the Montevideo criteria are considered core elements of international law, has convincingly demonstrated how the lack of effective government presented no impediment to the majority of western states, including the EC and the US, which recognized Bosnia as a state in 1992 despite ongoing armed conflict.12 It would appear then that the strict interpretation of government to mean total independence and effective control is unreasonable and justified neither by international law nor state practice.

The claim that Palestine does not have the capacity to enter into relations with other states is based upon provisions of the Oslo Accords severely restricting Palestinian ability to enter into foreign relations,13 and even where tolerated it was to be the PLO and not the PA who could exercise such capacity.14 Shaw argues that ‘critical functions seen as indispensible to statehood in international law have by agreement between the relevant parties been recognized as matters subject to Israeli control’.15

Nonetheless, despite the failure of the Oslo process to achieve the stated outcome of Palestinian independence, Palestine has emerged as an increasingly autonomous international actor in the economic, legal, and security spheres. It is clear that international practice is to overlook the Oslo restrictions for the benefit of the Palestinian people and to support their steps towards independence.16 As noted in the Al-Haq paper, the EU considers the PA evolved enough to disregard many of the Interim Agreement restrictions regarding its capacity to enter foreign relations. The Euro-Mediterranean Interim Association Agreement on trade and cooperation was signed between the European Community and the PLO (on behalf of the PA) in 1997.17 In 2005 the EU Coordinating Office for Palestinian Police Support was formally established by an exchange of letters between the PA and the EU Special Representative to the Middle East Peace Process, and it was on the explicit written invitation of the PA that the Council of the European Union established the EU Police Mission in the Palestinian Territories the same year under the European Security and Defence Policy.18

It would appear that Shaw’s conclusion that Palestine ‘simply cannot conform with the basic requirements in international law of statehood’ since the only powers and responsibilities that the PA may exercise are those that have been expressly transferred by Israel by way of Oslo,19 no longer – if ever – stood up to the realities of state practice, including the practice of the Arab League states, the EU, US and others.20

Although this may seem an assault on the principle of legality, the restrictions on Palestinian development which Oslo construed were intended to be temporary at which point Palestine would have reached independence. Given the incessant violation of Oslo by all parties, its limited temporal framework, and the extent to which international policy is committed to developing Palestinian institutions in preparation for independence, there can be little criticism of the trumping by Palestine of the exercise of its sovereign right to self-determination over the mean terms of Oslo. As the only sovereign in the oPt there is no basis in international law or practice to Shaw’s claim that by acting beyond the Oslo limitations Palestinian claims to statehood ‘may well be therefore vitiated by such illegal activity’.21

A Contrary Approach

As with Quigley’s submission, Mendes draws upon the Palestinian right to self-determination and to an independent state, the Palestinians’ monopoly over sovereignty in the oPt, Palestinian engagement with international institutions and relations with other states, and recognition by third states (as well as institutions and organisations), to conclude that Palestine is a state, denied full recognition and independence only by powerful geopolitical interests. Palestine can therefore engage fully and on an equal basis with other states with international institutions including the ICC. Mendes warns against a simple ‘mechanical application of the Montevideo Convention’ when considering statehood in light of ‘de facto states…under belligerent occupation or protected status or under some form of international administration’.22 In particular, Mendes undermines the correctness of uncritically accepting the Montevideo orthodoxy as constituting established or customary international law, identifying state practice as being closer to a constitutive theory of statehood, whereby recognition must be considered at least as significant as the four criteria. This is in keeping with James Crawford’s observation that ‘the formula represented in the Montevideo Convention is considered to a certain extent insufficient and outdated, even “hackneyed”.23 Both Quigley and Mendes make compelling arguments for the existence of the Palestinian state by a combination of the recognition of a Palestinian state and of Palestinian capacity to enter foreign relations, taken with an interpretation of Montevideo criteria that avoids a simple mechanical approach, and having reference to the Palestinian right to self-determination, sovereignty, and the inequity of occupation.

The Functional Approach

Mendes further considers whether the relevance of international legal personality, as opposed to supposed general criteria of statehood under international law, should be the test when considering the application of the Rome Statute to the Palestinian declaration, concluding that at the least Palestine enjoys adequate international legal personality to be regarded as a state for the purposes of article 12(3).24

This is the approach advanced by Al-Haq and Alain Pellet. It is striking that neither Al-Haq nor Pellet give any attention to Montevideo, contending that while it appears that Palestine does exhibit all the attributes of a state, the question for the Prosecutor is not to pronounce in abstractio on whether ‘in the absolute’ Palestine is or is not a state. Instead, by having reference to the rights, duties, and actions of Palestine and the international community, the OTP should acknowledge that Palestine can be considered a state for the purposes of article 12(3) of the Rome Statute. The essence of the approach advocated is that since the Rome Statute makes no reference to Montevideo, nor to the definition of ‘state’, the question at hand must be framed narrowly within the ‘precise and particular context of Article 12 of the Statute of the ICC25

In stressing that the role of the ICC is not to recognise a Palestinian state, a matter which remains the prerogative of individual states as well as international organisations, but to identify whether Palestine can be considered a state for the purposes of article 12 and therefore legitimately transfer its jurisdiction to the Court, Pellet advocates for the adoption by the OTP of a functional, ‘teleological and practical’, approach.26 Of course it goes without saying that were the Court to accept Palestine as a state for the purposes of the Rome Statute, it is unlikely that such a decision would not impact on the willingness or otherwise of many states to individually recognise Palestine, but since that remains their free choice such considerations, so far as the Court is concerned must remain by-the-by.

Pellet suggests the Prosecutor adopt a functional, practical approach on the basis of such being a common and well established feature of international law. Thus the UNGA in requesting an advisory opinion from the ICJ re Kosovo did not ask whether Kosovo is a state, but rather whether the unilateral declaration of independence conforms with international law.27 Jurisprudence from a variety of international treaties and tribunals, including the Convention on the Rights of People with Disabilities, the Convention on International Liability for Damage Caused by Space Objects, the European Court of Justice, European Court of Human Rights, and the International Centre for Settlement of Investment Disputes Tribunal, is cited in support of this approach.28

Having demonstrated the legitimacy of adopting a functional approach in understanding the concept of a state for the purpose of international treaties and international law, Pellet observes that ‘the context and the object and purpose of the Statute and of its Article 12 are of particular importance due to the “variable geometry” of the very concept of the State, which makes it difficult to keep to a single unambiguous meaning, and, therefore to an ‘ordinary meaning’. In addition, the determination of the jurisdiction of international bodies (organisations and courts – the ICC being both) is a privileged area of teleological treaty interpretation.’29 Drawing on article 31 of the Vienna Convention of Treaties, and the jurisprudence of the International Court of Justice and the International Criminal Tribunal for the Former Yugoslavia,30 Pellet reiterates that Palestine has the sole claim to sovereignty in the oPt, to conclude that the ICC has kompetenz-kompetenz to accept the Palestinian declaration as being valid and effective in accordance with the provisions of article 12 and that all conditions for the Court to exercise its jurisdiction in pursuance of Article 13 are met.31

Pellet’s analysis complements Al-Haq’s paper which demonstrates that since the PA has the capacity to enter into relations with states, and since it has been recognised by the international community as having the capacity to exercise jurisdiction over crimes within the jurisdiction of the Rome Statute , it must therefore be recognised as having the ability to transfer such jurisdiction to the ICC by means of an article 12(3) declaration.

Confirmation of Palestinian jurisdiction over international crimes arises from what may be one of the most significant, yet implicit, recognitions of Palestinian statehood yet. This relates to the extent of Palestinian criminal jurisdiction that could be transferred to the ICC. The PA exercises criminal jurisdiction over Palestinians and nationals of third states other than Israel in Area A of the oPt.32 Under Oslo it is clear that the Palestinian legal system has no jurisdiction, civil or criminal, over Israeli citizens. Al-Haq reported that in order to overcome this prima facie obstacle, the PA submission to the OTP argued that in concluding an interim, that is, a temporary agreement with Israel to exclude Israelis from the scope of their criminal jurisdiction in the West Bank and Gaza, the PA temporarily waived an inherent right which as the bearers of the right to self-determination and to an independent state in the West Bank and Gaza, they continue to hold.33 Such a waiver was conceived of along with many other compromises – such as the continued presence of unlawful settlements – not as acceptance of any Israeli claim or otherwise but as part of a bona fide effort to secure an end to the conflict and to see the two states living side by side. By this reckoning, the current situation vis-à-vis PA jurisdiction over Israelis may be reversed should they choose to unilaterally withdraw from the Accords. Such an interpretation by the PA would support the claim advanced earlier that the Palestinians have always considered on the basis of their uncontested sovereignty their state to have existed at least since the 1988 declaration even if it has not achieved independence.

Al-Haq suggested an additional, more persuasive argument by which to establish the requisite levels of capacity by reference to the customary nature of the grave breaches regime of international humanitarian law. Specifically, crimes over which the ICC has jurisdiction, including grave breaches of the Geneva Conventions, crimes against humanity, and genocide are crimes under customary international law, and many are widely recognised as being subject to universal jurisdiction. There is an obligation upon all states to enact effective penal sanctions in domestic law and an obligation to search for and to try or extradite persons suspected of grave breaches on the basis of universal jurisdiction, regardless of the nationality of the perpetrator. With 194 states parties, the Geneva Conventions of 1949 and the grave breaches regime, as confirmed in 1996 by the International Court of Justice in its advisory opinion on the threat or use of nuclear weapons, form part of customary international law.34

The grave breaches of the Geneva Conventions documented in the Goldstone Report are alleged to constitute war crimes and possibly crimes against humanity. The General Assembly resolution that endorsed the Report, urged: in line with the recommendation of the Fact-Finding Mission, the undertaking by the Palestinian side, within a period of three months, of investigations that are independent, credible and in conformity with international standards into the serious violations of international humanitarian and international human rights law reported by the Fact-Finding Mission, towards ensuring accountability and justice.35

This represents an acknowledgement by the international community that, whether there exists a Palestinian state or not, ‘the Palestinian side’ not only has the ability to investigate and prosecute international crimes but also that it has the duty to do so. Such investigations, to be in conformity with international standards must be directed against all those responsible for the alleged crimes and not discriminate on grounds of nationality. Article 146(2) of the Fourth Geneva Convention is explicit in stating that each High Contracting Party “shall be under the obligation to search for persons alleged to have committed, or to have ordered to be committed, such grave breaches, and shall bring such persons, regardless of their nationality, before its own courts”.36

It is impermissible however to create distinctions on the grounds of nationality in the duty to investigate and prosecute any individual allegedly responsible for grave breaches.37 The exclusion of Israelis from PA jurisdiction as provided for in the Interim Agreement cannot legitimately be considered as extending to the international crimes of war crimes and crimes against humanity as to do so would be incompatible with international law. As an entity acknowledged by the international community as having both the capacity and responsibility for investigating and prosecuting serious violations of international human rights and humanitarian law the PA must therefore be acknowledged as having the capacity and responsibility for investigating Israelis suspected of being responsible for such actions.

The Resolution, and statements from states that opposed its adoption but nonetheless called upon the Palestinian authorities to undertake investigations, was a clear assertion that the Palestinian authorities were recognised as having the capacity and jurisdiction to investigate and prosecute Grave Breaches of the Geneva Conventions. By this reckoning there is broad consensus that Palestine, at least when it comes to the application and enforcement of international humanitarian law, is on a par with ‘proper’ states.

Summary

What has emerged from the documents in the Summary of Arguments are two very different approaches to the question at hand – indeed at heart there is a division even as to what the question is that is being asked. I would submit that the Orthodox approach is not only unduly conservative and strict interpretation, but a wholly unreasonable reading and assertion of both general international law and the text of the Rome Statute itself. While there is certainly merit to the analysis proffered by Mendes and Quigley, given the purpose and principles of the International Criminal Court, the legitimacy and commonality of the functional approach as noted by Pellet, and the factual conditions of capacity and recognition pertaining to the extent of Palestinian jurisdiction, it would appear that the proper and lawful procedure to be followed by the OTP is to accept the transfer from Palestine of jurisdiction over the crimes set forth in the Statute.

One issue raised above demands some further consideration. Yuval Shany published a strong defence of the functional interpretation of article 12(3) of the Rome Statute, yet concluded that due to the terms of the Oslo Accords, neither the Court nor the Prosecutor can accept the Palestinian declaration since the Palestinians have waived their potential to engage in foreign relations.38 This deserves some additional consideration since it is repeatedly cited as a basis for rejecting the Palestinian engagement with the Court. In the first instance, and as discussed in the Al-Haq paper, international humanitarian law recognises that protected persons, those ‘who at any given moment and in any manner whatsoever, find themselves, in cases of a conflict or occupation in the hands of a Party to the conflict or Occupying Power of which they are not nationals’,39 are liable to come under immense pressure to forfeit the protections guaranteed them under the Fourth Geneva Convention.

The drafters were conscious of the experience during the Second World War when national governments struck agreements with occupying powers that were presented to those concerned ‘as an advantage, but in the majority of cases involved drawbacks which were sometimes very serious’,40 addressed this potentiality in Article 8 of the Fourth Geneva Convention which asserts that: protected persons may in no circumstances renounce in part or in entirety the rights secured to them by the present Convention and by the special agreements referred to in the foregoing Article.

The drafters also wanted to ensure that states could not take ‘refuge behind the will of the protected persons’ to justify their failure to comply with the provisions of the Convention. In this spirit, the drafters further emphasised the ‘cardinal importance’ of the non-derogability of the Convention’s protections, with Article 47 asserting that: Protected persons who are in occupied territory shall not be deprived, in any case or in any manner whatsoever, of the benefits of the present Convention by any change introduced, as the result of the occupation of a territory, into the institutions or government of the said territory, nor by any agreement concluded between the authorities of the occupied territories and the Occupying Power, nor by any annexation by the latter of the whole or part of the occupied territory.

In sum, Articles 8 and 47 affirm that belligerents cannot conclude agreements which derogate from or deny to protected persons the safeguards of the Fourth Geneva Convention. Nor can any renunciation of rights by protected persons have legal effect. On this basis it cannot be countenanced in law that the Oslo Interim Agreement can be regarded as today excluding from the jurisdiction of the PA the obligation set forth in Article 146 (2) to prosecute any individual allegedly responsible for grave breaches. Neither can the Palestinian right to self-determination, a jus cogens norm,41 one aspect of which is the ability to engage in international relations with other peoples,42 be denied. That Oslo does not provide the basis for excluding Palestine from engagement with the ICC is further accentuated by recent and ongoing Palestinian engagement at the International Court of Justice and at UN institutions such as the Human Rights Council and Security Council, all of which clearly constitute precisely those manifestations of foreign relations supposedly unlawful under Oslo. Sean Murphy noted that in its treatment of Palestine throughout the Advisory Opinion on the Wall proceedings, by allowing Palestine to make written and oral submissions and in much of the ius in bello analysis ‘the Court appears to regard Palestine as the functional equivalent as a state’.43 Similarly Quigley cites UN Security Council practice of permitting Palestine to routinely participate in sessions where relevant issues are on the agenda, despite Security Council rules providing that only a state is entitled to participate.44

These precedents support the contention that Palestine is for many purposes of international considered a state, despite not enjoying independence. More critically though, this bulwarks the argument that not only can and should the Prosecutor adopt a functional approach to Article 12(3), but that the terms of the Oslo Accords can in no way be considered as denying Palestine the right to engage with the International Criminal Court.

In conclusion, this author sees in Palestine many, if not all, the orthodox characteristics of statehood, and stresses the underlying sovereignty and right to self-determination of the Palestinian over the oPt. Nonetheless, limiting the question of what constitutes a state to an abstract question is cautioned against. By looking to the purpose of the Rome Statute and relying upon the recognition that Palestine has jurisdiction over the crimes set forth in the Rome Statute, it is submitted that the best approach is a functional approach, and the acknowledgment that what is at question is not the issue of whether Palestine is a state or not. Having had regard to the precedents of other international tribunals when faced with the question of the meaning of state – a term whose meaning remains subjective and unbound from any definitive criteria – and a contextual interpretation of the Rome Statute, and noting the recognition by the international community that Palestine shares with all other states the duty to investigate and prosecute Grave Breaches of the Geneva Conventions, it is submitted that the proper and lawful procedure to be followed by the OTP in this situation is to accept the transfer from Palestine of jurisdiction over the crimes set forth in the Statute.

Endnotes — (click the footnote reference number, or ↩ symbol, to return to location in text).

  1. 1.

    Robert Weston Ashe Is Palestine a State”? A Response to Professor John Quigley’s Article, “The Palestine Declaration to the International Criminal Court: The Statehood Issue, 36 Rutgers Law Record (2009) [hereinafter cited as Statehood Issue], 187.

  2. 2.

    ECLJ, Legal Memorandum in Opposition to the Palestinian Authority’s January 2009 Attempt to Accede to ICC Jurisdiction Over Alleged Acts Committed on Palestinian Territory Since 1 July 2002, 9 September 2009 [hereinafter cited as Opposition Memo], 12; Yaël Ronen, ICC Jurisdiction over acts committed in the Gaza Strip: art 12(3) of the ICC Statute and non-state entities, 8 J. Int’l Crim. Just. 1 (2010) [hereinafter cited as ICC Jurisdiction], 3–27, 5.

  3. 3.

    IAJLJ Opinion of Malcolm Shaw: In the Matter of the Jurisdiction of the International Criminal Court with regard to the Declaration of the Palestinian Authority, 9 September 2009 [hereinafter cited as Shaw Opinion], 21–2.

  4. 4.

    ECLJ, Opposition Memo, supra note 2, 14; Daniel Benoliel & Ronen Perry, Israel, Palestine and the ICC, Michigan Journal of International Law, Vol. 32, p. 73, 2010, available online. David Davenport, The Palestinian Declaration and ICC Jurisdiction, 19 November 2009 [hereinafter cited as Palestinian Declaration], 8–9. Ronen, ICC Jurisdiction, supra note 2, 6.

  5. 5.

    Ma’an News Network, Erekat: Time for countries to recognize Palestine 30 May 2010. Available online.

  6. 6.

    Pact of the League of Arab States, Annex on Palestine. Cited by Ronen, ICC Jurisdiction, supra note 2, 11.

  7. 7.

    Shaw Opinion, supra note 3, 27–31.

  8. 8.

    Convention on the Rights and Duties of States, 26 December 1933, 49 stat 3097.

  9. 9.

    ECLJ, Opposition Memo, supra note 2, 18; Shaw Opinion, supra note 3, 11,

  10. 10.

    Davenport, Palestinian Declaration, supra note 4, 9–11. ECLJ, Opposition Memo, supra note 2, 19.

  11. 11.

    Errol Mendes, Statehood and Palestine for the Purposes of Article 12(3) of the ICC Statute: A Contrary Perspective, 25.

  12. 12.

    Id. at 18.

  13. 13.

    Shaw Opinion, supra note 3, 15.

  14. 14.

    ECLJ, Opposition Memo, supra note 2, 19.

  15. 15.

    Shaw Opinion, supra note 3, 35.

  16. 16.

    Michael Kearney & Stijn Denayer, Al-Haq Position Paper on Issues Arising from the Palestinian Authority’s Submission of a Declaration to the Prosecutor of the International Criminal Court under Article 12(3) of the Rome Statute, 14 December 2009 [hereinafter cited as Al-Haq], ¶ 27.

  17. 17.

    Official Journal of the European Communities, 16.7.97, No L 187/3.

  18. 18.

    Council Joint Action 2005/797/CFSP of 14 November 2005 on the European Union Police Mission for the Palestinian Territories.

  19. 19.

    Shaw Opinion, supra note 3, 16.

  20. 20.

    Al-Haq, supra note 16, at ¶¶ 25–28.

  21. 21.

    Shaw Opinion, supra note 3, 35; Mendes, supra note 11, 22–26, 29.

  22. 22.

    Mendes, supra note 11, 2.

  23. 23.

    James Crawford, The Creation of States in International Law, (Oxford University Press, 2007, 2nd ed) 437. Mendes also brings to attention the unusual situation whereby a very significant number of claims as to the law of statehood made by those advocating the rejection of the Palestinian declaration and adoption of a strict interpretation of the Montevideo criteria almost exclusively rely upon the Crawford’s writings on the topic. With all due respect to Crawford’s scholarship, such overwhelming reliance on a single scholar, particularly in light of the statement quoted above, demands serious questioning of the arguments and reasoning upon which the Orthodox approach is premised. Mendes, supra note 11, 27.

  24. 24.

    Mendes, supra note 11, 44–8.

  25. 25.

    Alain Pellet, Effects of Palestinian Recognition of the Jurisdiction of the ICC, 14 February 2010 ¶ 5 (original in French); Mendes, supra note 11, 38; Al-Haq, supra note 16, ¶ 43.

  26. 26.

    Id. at ¶ 4.

  27. 27.

    Id. at ¶ 7.

  28. 28.

    Id. at ¶¶ 8–15.

  29. 29.

    Id. at ¶ 18.

  30. 30.

    Id. at ¶¶ 17–23.

  31. 31.

    Id. at ¶ 33.

  32. 32.

    Al-Haq, supra note 16, ¶ 30.

  33. 33.

    Id. at ¶ 32.

  34. 34.

    Jean-Marie Henckaerts, The grave breaches regime as customary international law, 7 J. Int’l Crim. Just. 4 (2009) 697.

  35. 35.

    Al-Haq, supra note 16, ¶ 34.

  36. 36.

    Id. at ¶ 34.

  37. 37.

    Id. at ¶ 36.

  38. 38.

    Shany, Yuval, In Defence of Functional Interpretation of Article 12(3) of the Rome Statute: A Response to Yaël Ronen, 8 J. Int’l Crim. Just. 2 (2010) 329–343.

  39. 39.

    Article 4, Fourth Geneva Convention. This provision extends to all who are not of the nationality of the occupying state: Pictet Commentary to Geneva Convention IV, 46, n 4. HSRC Occupation, Colonialism, Apartheid? A Reassessment of Israel’s practices in the occupied Palestinian territories under international law, (Cape Town, 2009) 71–81, available online.

  40. 40.

    Pictet, supra note 39, 69–70.

  41. 41.

    Antonio Cassese, International Law, (Oxford, 2nd ed 2005) 65; Martin Dixon, International Law, (Oxford, 6th ed 2007) 164.

  42. 42.

    Geoffrey Watson, The Oslo Accords: International Law and the Israeli-Palestinian Peace Agreements, (Oxford University Press, 2000) 245.

  43. 43.

    Sean D. Murphy, Self-Defense and the Israeli Wall: Advisory Opinion on Construction of a Wall in the Occupied Palestinian Territory, 99 Am. J. Int’l L. 92 (2005) 63. Cited by Mendes, supra note 11, 46.

  44. 44.

    Statehood Issue>, supra note 1, 5.

  45. Suggested Citation for this Comment:

    Michael Kearney, It is Imperative that in Considering the Issue at Hand, the Correct Question be Asked, and that Palestinian Jurisdiction Over Crimes Set Out in the Rome Statute be Acknowledged as Giving the ICC the Ability to Accept the PA Declaration, ICC Forum (Sep. 1, 2010), available at https://iccforum.com/gaza#Kearney.

    Suggested Citation for this Issue Generally:

    Should the ICC Investigate War Crimes in Gaza?, ICC Forum (Sep. 1, 2010), available at https://iccforum.com/gaza.

John Quigley Avatar Image John Quigley, M.A., LL.B. President’s Club Professor in Law Moritz College of Law, The Ohio State University

The ICC prosecutor has authority to investigate, and the ICC has jurisdiction, because the prosecutor has in hand a consent declaration from the state on whose territory the alleged crimes were committed.

Palestine has been a state since Turkey renounced sovereignty over its territories in the Arab world after World War I.

Summary

The Palestine declaration of January 21, 2009 confers jurisdiction on the International Criminal Court for crimes committed on Palestine territory. The declaration was submitted by the state on whose territory crimes within the subject matter and temporal jurisdiction of the Court were allegedly committed. That state is Palestine.

The Court may be able to find jurisdiction over crimes allegedly committed in connection with the 2008–09 Gaza hostilities on the basis of the Palestine declaration without reaching the question of whether Palestine is a state. But one certain basis for a finding of jurisdiction is Palestine’s status as a state.

Palestine has been a state since Turkey renounced sovereignty over its territories in the Arab world after World War I. The Treaty of Lausanne, signed by Turkey and the World War I Allies in 1923, provided that Turkey’s Arab territories would be constituted as states. That same disposition was foreseen for Turkey’s Arab territories by the League of Nations in its 1919 Covenant. In that document, the independence of these territories was provisionally recognized. A new international institution was devised for the purpose, denominated mandate. Five states emerged—Iraq, Syria, Lebanon, Palestine, and Transjordan. Great Britain was to be a “mandatory” responsible for Palestine as a “mandate territory.” The Treaty of Lausanne made clear that even pending independence, the mandate territories were states.

Great Britain assumed administration of Palestine on this understanding. The statehood of the five territories was acknowledged in the international community. Since their status was established by the League Covenant, all League members can be said to have participated in the project. States that were not members of the League acquiesced in the status arrangement. No state objected. Many states concluded treaties with these new states, and in particular with Palestine. All states for whom the issue arose acknowledged the citizenship that these states attributed to their inhabitants. As regards Palestine, when issues arose in litigation, or potential litigation, that brought into question its status, the outcome uniformly was that Palestine was a state.

Palestine statehood survived Britain’s withdrawal in 1948. The Charter of the United Nations provided for the preservation of the rights of both states and peoples under existing international arrangements, which included the mandate arrangements. Palestine’s statehood has since been recognized by the main organs of the United Nations as issues have arisen relating to its status. Palestine is recognized diplomatically by over one hundred states. In the Middle East peace process, Palestine is viewed by the international community as an entity entitled to negotiate borders and to recognize other states, in particular to recognize Israel. Negotiation of borders and recognition of states are acts undertaken only by states. Through the peace process, Palestine has been implicitly recognized as a state.

Argument

The military incursion by Israel into the Gaza Strip in 2008–2009 generated allegations of crimes potentially within the jurisdiction of the International Criminal Court (ICC). The Office of the Prosecutor at the ICC was presented with information from non-governmental organizations about such violations, with the suggestion that the Office investigate. Under the multilateral treaty creating the court, a document titled “Statute,” the ICC has subject-matter jurisdiction over war crimes, crimes against humanity, and genocide. It has temporal jurisdiction over such acts if committed after July 1, 2002.

Under the ICC Statute, jurisdiction is based largely on adherence by states as state parties to the Statute. Unless the UN Security Council refers a situation—which has not occurred in regard to the Gaza hostilities—the ICC has jurisdiction only if the act, even if falling within subject matter and temporal jurisdiction, is related to a particular state. That relation can be in one of four ways. The first is that the act was committed in the territory of a state party to the Statute. The second is that the act was committed by a person who is a national of a state party. The third is that the act was committed by a national of a non-party state, but where that state files a declaration with the Court conferring jurisdiction. The fourth is that the act was committed on the territory of a non-party state, but where that state files a declaration with the Court conferring jurisdiction.

As regards Israel’s incursion of 2008–2009 into the Gaza Strip, the first route to jurisdiction was lacking because Palestine was not a party to the Statute. The second route was, for the most part, lacking because Israel as well was not a party, so individuals holding Israel’s or Palestine’s nationality would not fall under the Court’s jurisdiction on the basis of their nationality.

On January 21, 2009, a declaration was filed with the Registrar of the ICC in the name of Palestine. The declaration did not confer jurisdiction for acts by nationals of Palestine, but it did confer jurisdiction for acts after July 1, 2002 and falling within the Court’s subject matter jurisdiction, if they were committed on the territory of Palestine.

The declaration’s validity in conferring that scope of jurisdiction turned on construction of the Statute, as well as on the status of Palestine as an international entity. A declaration of this type is to be filed by “the State on the territory of which the conduct in question occurred.” The Statute does not define “territory” or “state.” As specified in Article 31 of the Vienna Convention on the Law of Treaties, a document widely regarded as reflecting the customary law of nations in regard to the construction of treaties, terms are construed in their “ordinary meaning,” taken “in their context” and in light of the “object and purpose” of the treaty. For “territory,” this probably means an act committed not only by a person who is physically on the territory, but also an act committed by a person who is physically outside the territory but where the act produces effects on the territory.

Article 31 opens the possibility of a construction of the term “state” to include entities that might not fit under the “ordinary meaning” of “state,” given the treaty’s “object and purpose” of protecting against atrocities. But even without an expansive reading, the term “state” as understood in general international law includes Palestine, in whose name the declaration of January 21, 2009 was filed.

Ordinary meaning of the term “state”

In international usage, the term “state” covers a broad range of territorial entities. At one extreme, in terms of territory or population level, stand such entities as China, the Russian Federation, and India. At the other extreme stand so-called “micro-states,” which have minimal territory and population. The fact that micro-states are states means that they are included in the ordinary meaning of the term “state.” Thus, for example, the State of Niue, a grouping of islands off New Zealand that formerly was under New Zealand’s trusteeship, has been admitted as a member of a number of UN-related organizations that require statehood for membership. Niue has been ruled by the UN Secretary-General to qualify as a “state” when it has acceded to multilateral treaties to which only states may be party. Niue’s land area is 260 square kilometers. Niue’s inhabitants number fewer than two thousand, and they are nationals of New Zealand, because Niue has no nationality of its own. Niue’s status is not recognized by any international document, but rather by a piece of New Zealand legislation.

Beyond territory and population, a state normally has a government with some minimal level of control over its affairs. The level of control required, however, is not high. Entities whose foreign affairs are handled by an outside state nonetheless qualify as states. Niue has no diplomatic service or embassies, these functions being handled on Niue’s behalf by New Zealand. Responsibility for national defense may rest wholly in the hands of an outside state. Palau, another grouping of islands in the Pacific Ocean, was formerly a U.S. trust territory. By bilateral treaty, the United States has complete authority over the defense of Palau, to the exclusion any authority held by Palau itself. Yet Palau has been admitted to membership in the United Nations, an organization that requires statehood for membership.

With some micro-states, an outside state plays a decisive role even in internal governance. France plays such a role in Monaco. By bilateral treaty, seconded French civil servants head a number of governmental departments in Monaco. Yet Monaco, like Palau, is a member state of the United Nations.

The criterion of control vanishes altogether if a state’s territory is held by another state in belligerent occupation. A state whose territory has been occupied by a foreign army may exercise little or no control. Yet its statehood is not thereby negated.

In one other category of “state,” internal control is lacking. In states commonly referred to as “failed,” the government may exercise little control over sectors of the territory, or perhaps no control anywhere in the territory. Yet such entities are considered to be states.

The Convention on Rights and Duties of States (Montevideo) of 1933 is often cited for a listing of the criteria required for statehood. The Convention lists, prominently, territory, a population, and control by a government over the population in the territory. As indicated, these criteria are honored more in the breach in the contemporary practice of states.

Status of Palestine as a Class A mandate territory

It is sometimes said that in order to be a state, an entity must be independent of other states, but this proposition is doubtful. The “independence” of some of the micro-states is fragile at best. The absence of a requirement of independence is demonstrated by the status in the aftermath of World War I of Palestine and the other formerly Ottoman territories. Iraq, Syria, Lebanon, Palestine, and Transjordan were set up as states under a newly devised system called mandates, that involved “tutelage” by an outside state on a temporary basis. Supervision was to be exercised by the League of Nations, through a special agency it created for the purpose, called the Permanent Mandates Commission. The outside power was called a “mandatory.” In the case of Palestine, Great Britain held the status of mandatory and was given the power of administration.

These territories came to be called “Class A” mandates, to distinguish them from other mandates whose international status was less than statehood. The Class A mandates were, according to the League of Nations Covenant, “provisionally recognized as independent nations.” The 1923 Treaty of Lausanne, which embodied the terms of peace with Turkey, referred to these territories as “states” being “detached” from Turkey. The territorial disposition effected by the Treaty of Lausanne was accepted by the international community. There were differences among the five in the relationship between the territory and the mandatory power, but all were regarded by the League, and by its Permanent Mandates Commission, as holding the same status, namely, statehood.

As is more fully recounted in my The Statehood of Palestine: International Law in the Middle East Conflict, to be published shortly by Cambridge University Press, Palestine functioned as a state under this arrangement and was regarded as a state by other states. Palestine’s inhabitants held Palestine nationality, pursuant to a provision of the Treaty of Lausanne that called for a Palestine nationality. That nationality was recognized by other governments, which honored Palestine passports. When issues of the nationality of Palestine’s inhabitants arose in foreign courts, Palestine nationality was routinely accepted.

Palestine entered into bilateral and multilateral treaties with states in the immediate region and with European states. These treaties were concluded by Britain’s High Commissioner as the executive authority in Palestine. Under the mandate arrangement it was Britain who held the power to conclude treaties for Palestine. A distinction is made in regard to treaty-making between power and capacity. For Palestine, Britain exercised the power, but the capacity was that of Palestine.

When issues involving Palestine’s status surfaced in practice at the international level, Palestine was found to be a state. In 1932, for example, after the British Parliament enacted a general tariff law, the British Government contemplated extending a tariff preference to Palestine so that goods entering Britain from Palestine would not be subject to duty. But Britain had most-favored-nation clauses in bilateral commercial treaties with a number of other states. If it offered a tariff preference to Palestine, other states could claim that goods from their territories could be assessed tariffs by Britain no higher than Britain collected on goods from Palestine.

Rather than grant Palestine a preference and run that risk, the British Government decided to survey Italy, Spain, and the United States, with each of whom it had a most-favored-nation treaty. It asked them whether, if Britain were to grant a preference to Palestine, they would claim it as well. The three countries said they would, their rationale being that Palestine was a state. The British government contemplated suing the three states over the issue in the Permanent Court of International Justice. But Britain’s Attorney General advised that Italy, Spain and the United States would likely be ruled to be validly invoking the most-favored-nation provisions on the rationale that Palestine was a state. Britain dropped the plan to sue, or to grant Palestine a trade preference.

The academic lawyers of the League era found the status of the mandate territories a source of fascination, and of confusion. Dissertations were written on the topic in Vienna and Paris. Articles filled the legal periodicals. The academics were not of one mind on the status of the Class A mandates. But the governments of the era regarded them as states and dealt with them in that way.

The top British-appointed officials in Palestine itself regarded Palestine as a state. Britain’s High Commissioner, Sir Herbert Samuel said that Palestine under the mandate arrangement was a state, as did Norman Bentwich, who served as Attorney General of Palestine. The chair of the League of Nations Permanent Mandates Commission, which supervised Britain’s administration of Palestine, was of the same view. The fact that administration was exercised by Great Britain, and that Palestine’s independence was only “provisionally recognized” did not detract from the fact that Palestine was a state.

Status of Palestine after 1948

When a new international organization of states was formed, a clause was written into its Charter to preserve rights under League of Nations mandates. Article 80 of the Charter of the United Nations protected the rights of “states” or “peoples” under “existing international instruments to which Members of the United Nations may respectively be parties.” Britain, as party to the arrangement with the League of Nations for a mandate in Palestine, fell under this provision. Hence the international recognition of Palestine statehood did not vanish upon the demise of the League of Nations. Nor did it vanish when Britain withdrew in 1948. The rights of both Palestine as a state and of the people of Palestine were preserved by UN Charter Article 80.

In the wake of Britain’s withdrawal, a National Council took control of the greater portion of Palestine’s territory in the name of the Jewish community of Palestine. The National Council seceded from Palestine to form a state called Israel. Egypt assumed administration of the Gaza Strip, and Jordan (renamed from “Transjordan”) assumed administration in what at the time was called central Palestine, and soon to be called the West Bank of the Jordan River. Egypt and Jordan, each in its own way, acknowledged Palestine’s continuing statehood, consistent with UN Charter Article 80. Each put itself in a caretaker role pending the reemergence of a government to administer Palestine. The international community accepted the continuation of Palestine statehood by acquiescing in Egypt’s mode of administration in the Gaza Strip, and, though with some hesitation, in Jordan’s mode of administration in the West Bank of the Jordan River.

Since 1967, the territory over which Egypt and Jordan assumed administration in 1948 has been occupied by Israel under a regime of belligerent occupation. The control thus exercised by Israel had no impact on the statehood of Palestine. Although Palestine is deprived of control in its territory, its statehood subsists. This disposition is consistent with the character of belligerent occupation, which effects no change in the statehood of territory. Just as, for example, Denmark retained its character as a state while under occupation by Germany (1943 to1945), so Palestine retained its character as a state under occupation by Israel.

In 1988, a government-in-exile for Palestine was proclaimed. A declaration issued in that year by the Palestine Liberation Organization referenced the League of Nations Covenant and the Treaty of Lausanne on Palestine’s status in re-affirming Palestine statehood. That declaration was met with a resolution of the UN General Assembly welcoming this re-affirmation of Palestine statehood, and Palestine was shortly recognized in one way or another at the diplomatic level by over one hundred states. Palestine applied for membership in several specialized agencies affiliated with the United Nations. These organizations admit only states as members. Most member states favored Palestine’s admission, but the organizations decided not to act on the applications in light of threats of financial sanctions against the organizations issued by the Government and Congress of the United States.

Palestine in the contemporary world

The United States and the Soviet Union initiated a peace process three years later, calling on Palestine and Israel to come to terms in regard to a territorial divide. In 1993, the two parties penned an agreement in which they pledged to resolve the question of their borders. Benjamin Netanyahu, at the time the leader of the opposition in the Israeli parliament, denounced the agreement because, he said, it meant that Israel had recognized Palestine as a state. Netanyahu was correct in that assessment, because the agreement to negotiate borders presumed the statehood of each of the parties. Further, the Israeli government demanded the recognition of Israel, which Yassir Arafat as Chairman of the Palestine Liberation Organization extended in writing. Recognition, like the negotiation of borders, is an act in which only states engage. By demanding recognition of itself, Israel indicated that it regarded Palestine as a state.

Achieving an agreement over borders proved elusive, but the major powers pressed the parties to pursue the effort. This pressure from the international community showed that it, like Israel, regarded Palestine as a state. Recognition of Palestine statehood is reflected in particular in the Performance-based Road Map to a Permanent Two-State Solution to the Israeli-Palestinian Conflict, drawn up by the so-called Quartet (United States, Russia, European Union, and United Nations). Issued on April 30, 2003, the Road Map called on the major powers to promote recognition of Palestine in what was identified as Phase 2 of the Road Map, which was to begin in June 2003. Although Phase 2 was not reached, the issuance of such a plan, contemplating diplomatic recognition of Palestine within a few weeks, indicated that Palestine was regarded as a state.

In international practice, recognition of a state may be manifested in a variety of ways. Recognition need not be explicit. It may be implied from practice. The practice of states in the peace process shows that even those states that have not recognized Palestine formally nonetheless regard it as a state.

Although Palestine has yet to be admitted to membership in the United Nations, the main UN organs treat it as a state. The General Assembly accords to Palestine’s representatives the right to reply to statements made by representatives of other states, a right enjoyed, under General Assembly rules, only by representatives of states. The Security Council admits Palestine participation in its debates, a right accorded, under Security Council rules, only to member states. The Economic and Social Council, another of the main organs of the United Nations, treated Palestine as a state by admitting it, as it did, as a member of the Economic Commission for Western Asia, a body open only to states. The International Court of Justice also treats Palestine as a state. The Court invited Palestine to participate when it held proceedings leading to a 2004 advisory opinion about the barrier Israel was constructing at the time in the West Bank of the Jordan River. Under Article 66 of the Court’s Statute, only states may so participate.

Among the minority of states that have yet to accord Palestine formal diplomatic recognition are states of Western Europe. Their reticence, however, has nothing to do with their view about Palestine as a state. They withhold formal recognition in order to pressure Palestine to come to terms with Israel. Their view of the status of Palestine was made clear in 1997, when the European Community signed with the PLO the Euro-Mediterranean Interim Association Agreement on Trade and Cooperation. Under the founding treaty of the European Community, such agreements are concluded only with states located outside Europe. Hence, penning this agreement with the PLO meant that the European Community found the PLO to represent a state.

Palestine thus holds a status as a state that dates from the time of the League of Nations. Palestine falls comfortably within the meaning of the term “state” as that term is used in international practice, hence as one finds it in the ICC Statute. As a result, the Palestine declaration of January 21, 2009 is an instrument with legal effect. Since the Gaza Strip is part of the territory of Palestine, the ICC has jurisdiction in particular over offenses committed in relation to the hostilities of 2008–2009.

Counter-arguments

A number of organizations and individuals sent submissions to the Office of the Prosecutor to urge either the acceptance or rejection of the Palestine declaration. In May 2010, the Office posted on its website a summary of these submissions and their texts. Of these submissions, four argued against the validity of the Palestine declaration on the ground that Palestine is not a state. I submitted two of the items that were posted, presenting arguments along the lines of those above that Palestine is a state. One of the four submissions in opposition—submitted by two professors at the University of Haifa—was structured as a response to my arguments. The other three also took positions contrary to mine.

The arguments made against Palestine statehood in the four submissions ignored relevant facts and mis-construed principles of law. Several of the submissions cited statements by Palestine officials who spoke of establishing an independent Palestine state. These statements were taken by the authors of the submissions to mean that the officials regarded a state as coming into being in the future, but not as one that presently exists. Uniformly, however, the quoted statements were calls for an end to Israel’s occupation of Palestine territory. The purport of the statements was that the independence of the Palestine state was being infringed by Israel.

Several of the submissions pointed to statements in resolutions of the UN General Assembly and UN Security Council, and in the 2003 Road Map, advocating a two-state solution. These statements were taken as reflective of an understanding that Palestine is not presently a state. These submissions ignored the actions, cited above, by the General Assembly and Security Council accepting Palestine’s participation on the basis of its being a state. A call for a two-state solution, as made in the Road Map, does not presume the present non-statehood of Palestine. It is rather a way of calling for peaceful relations.

Several of the submissions cited the deferrals in 1989 by UNESCO and the World Health Organization of Palestine’s applications for membership as indications that the international community does not regard Palestine as a state. These submissions failed to mention the threats of financial reprisal, as noted above, that precipitated the deferrals.

Several of the submissions mistakenly referred to the Palestinian National Authority as the entity their authors said was not a state. The PNA is a governing authority for the state. It need not appear that the PNA controls Gaza for Palestine to be a state.

One of the submissions urging the invalidity of the Palestine declaration was sent by the Israel-based International Association of Jewish Lawyers and Jurists, which attached an opinion letter from Professor Malcolm Shaw QC. Shaw focused on the current Gaza-West Bank split and claimed that the control criterion for statehood was not met because of the fact that from 2007 the Palestinian National Authority did not control Gaza. But Shaw did not explain why a split of authority in two parts of Palestine territory reflects negatively on statehood. Other states—Vietnam after 1954 being an example—have had split administrations yet were states.

With regard to the level of control exercised, Shaw argued that Palestinian administrative powers are limited by the post-Oslo agreements. Shaw cited a clause in the 1995 Interim Agreement (Israel-PLO) that says that the PNA is not to exchange ambassadors with other states or conduct diplomatic relations. Shaw overlooked the fact that it is the PLO that conducts diplomatic relations for Palestine. Palestine’s foreign relations are conducted primarily by the PLO, not by the PNA.

Shaw also pointed to various ways in which the PNA lacks full control, for example, that under the post-Oslo agreements Israel controls the airspace. Shaw omitted the fact that Palestine territory is under belligerent occupation, a circumstance that limits its ability to exercise control.

Shaw wrote that any assertion of statehood, as reflected in the Palestine Declaration to the ICC, would violate a provision of the 1995 Interim Agreement stipulating that, pending agreement between them, neither side shall change the status of Gaza or the West Bank. Shaw ignored a provision of the same Interim Agreement that says that nothing in the Interim Agreement precludes either party from maintaining its positions on any issue. Given that Palestine statehood was asserted prior to 1995, an assertion of Palestine statehood after that date is no violation.

Another of the four submissions was on the letterhead of the Hoover Institution at Stanford University, signed by two fellows of the Institution and four international law professors. In one of its main points, the Hoover letter focused on the invitation to Palestine by the International Court of Justice, as mentioned above, to participate in its proceedings in the advisory opinion case concerning the construction of Israel’s West Bank barrier. The Hoover letter argued that this invitation did not imply Palestine statehood. It said that in contentious proceedings in the Court (a case by one state against another) only states participate, but that in advisory proceedings the Court, under its Statute, may, under Article 66 of its Statute, invite not only states but “international organizations.”

The Hoover letter supplied no documentation but implied that the “international organizations” could be non-state entities. That implication has no basis. The term “international organizations” as used in Article 66 refers to organizations of states. The Court does not invite NGOs. Moreover, even if the Court did invite NGOs, Palestine could hardly be characterized as an “international organization.” The invitation to Palestine could have been issued only under the category of “state.”

The Hoover letter made one other attempt to negate the Court’s acceptance of Palestine as a state. The letter recited: “Although Palestine was allowed to participate in arguments before the ICJ, it was not as a State. Indeed the court referred to its observer status at the United Nations.” The Hoover letter thus sought to juxtapose “state” to “observer,” implying that the Court, by calling Palestine a UN “observer,” put it in a category other than “state.” However, the Court’s reference to Palestine as an observer did not reflect a view that it was not a state. The Court referred to Palestine’s observer status, along with its having co-sponsored the resolution requesting an advisory opinion, as a way of providing a rationale for its invitation to Palestine. The Court did not refer to Palestine as an observer in order to distinguish it from a state.

On another point, the Hoover letter challenged the validity of the Palestine declaration to the ICC on the basis that it was filed by the PNA, whereas the entity that represents the Palestinian people at the international level is the PLO. This argument lacks merit. In the 1988 Palestine declaration, the PLO was declared to be the government of Palestine. The PNA was set up by the PLO in 1993 to administer territory on the basis of the Oslo agreements. It matters not which one submits a document of this sort. The declaration to the ICC implicates issues that the PNA, in its governing role, might handle, such as the surrendering of suspects to the court. It would be the PNA that would need to interact with the ICC at a practical level.

Daniel Benoliel and Ronen Perry, two members of the law faculty at the University of Haifa, submitted a draft law review article that, as originally posted by the Office of the Prosecutor, carried a notation that it was to be published in the Boston University International Law Journal, although that notation was later removed. A large segment of this draft article is devoted to criticizing points made in an article of mine that was published in the Rutgers Law Record (‘The Palestine Declaration to the International Criminal Court: The Statehood Issue,’ Spring 2009). Arguing against Palestine statehood, Benoliel and Perry cited the fact, which is true, that most western European states did not recognize Palestine in the wake of the 1988 Palestine declaration. They took this omission as an indication of Palestine non-statehood. They recited, as is also true, the fact that some of the states that declined to recognize “did so on the grounds that they wanted a more definite indication of Palestine’s positive attitude towards Israel, such as an explicit act of recognition of Israel.” Benoliel and Perry missed the implication of that view of the European states. If a state declines to recognize a particular putative state but says that it will do so if the putative state takes an action that could be taken at any time, then the former necessarily regards the entity as a state. Otherwise, it would make no sense to say that it will recognize upon performance of the desired action.

Benoliel and Perry sought to refute my argument that the statehood declared for Palestine in 1988 was not of a new state, but of a state that already existed. Benoliel and Perry disputed my position that Palestine, as a Class A mandate under the League of Nations, had been constituted as a state. However, they gave no documentation for their view and failed to reference any state practice of that era in relation to Palestine. Their only contention in support of their view related to the ethnic aspect of the Palestine mandate. They said that Palestine was being “reserved exclusively for the Jewish people to establish a future independent state,” as if that meant that it was not at that time a state. That characterization of the Palestine mandate is inaccurate, given that the Balfour Declaration of 1917, as incorporated into the Palestine mandate, called for protection of the existing population, which overwhelmingly was not Jewish. But even if their characterization were accurate, it missed the point that Palestine’s status in the mandate period had nothing to do with the ethnic question. What is critical is that the states of the international community regarded Palestine under mandate as a state and dealt with it as such in their interactions with it.

In sum, the four submissions contain no valid points to refute Palestine statehood. The weakness of the counter-arguments only reinforces the conclusion that Palestine is a state.

  1. Suggested Citation for this Comment:

    John Quigley, The ICC Prosecutor has Authority to Investigate, and the ICC has Jurisdiction, Because the Prosecutor has in Hand a Consent Declaration from the State on Whose Territory the Alleged Crimes were Committed, ICC Forum (Sep. 1, 2010), available at https://iccforum.com/gaza#Quigley.

    Suggested Citation for this Issue Generally:

    Should the ICC Investigate War Crimes in Gaza?, ICC Forum (Sep. 1, 2010), available at https://iccforum.com/gaza.

Yael Ronen Avatar Image Yaël Ronen Assistant Professor Sha’arei Mishpat College, Hod Hasharon, Israel

Admission of the Palestinian declaration would require the ICC Prosecutor to engage in questions of statehood and recognition, either generally or for the purposes of the ICC Statute. The ICC Prosecutor should avoid engaging in such questions.

The ICC is not and should not be regarded as a panacea. Its limits should not be disregarded, as this would jeopardise the Court’s legitimacy and effectiveness.

Summary

This article concerns three issues regarding the admissibility of the 2009 Palestinian declaration, all of which are linked to the question of statehood. It first argues that the ICC Prosecutor may not assume the existence of a Palestinian state because the Palestinians themselves do not make a claim to that effect. It then rejects a purposive interpretation of Article 12(3) according to which declarations should also be admitted from quasi-states. Finally, it posits that admitting the Palestinian Declaration would embroil the ICC Prosecutor in undesirable conflicts over statehood and recognition.

Argument

1. Introduction

In December 2008–January 2009 Israel carried out a three-week long military offensive against Hamas in the Gaza Strip (‘Operation Cast Lead’). The conflict was followed by numerous allegations of violations of the laws of armed conflict on the part of both Israel and Hamas,1 and by calls to determine and enforce the responsibility for these violations, of state actors, non-state entities and individuals.2 Most prominently, a United Nations Fact Finding Mission on the Gaza Conflict appointed by the Human Rights Council3 concluded in a September 2009 report (the ‘Goldstone Report’) that both the Israeli military and Palestinian armed groups had violated international humanitarian law by indiscriminately and intentionally targeting civilians.4 Of the various enforcement mechanisms capable of responding to violations of the laws of war, international criminal law and recourse to the ICC specifically have been most forcefully advocated. The Goldstone Report called on the Security Council to demand of both Israel and the relevant authorities in the Gaza Strip to conduct good faith investigations that are independent and in conformity with international standards, and, if those are not undertaken, to refer the situation in Gaza to the ICC Prosecutor.5 It also recommended invocation of state responsibility pursuant to the Fourth Geneva Convention. The Human Rights Council has endorsed the Mission’s recommendations and called upon all concerned parties, including United Nations bodies, to ensure their implementation.6

On 21 January 2008 the Palestinian Minister of Justice lodged with the ICC Registrar a ‘Declaration Recognizing the Jurisdiction of the International Criminal Court’.7 The declaration, signed by the ‘Government of Palestine’ on ‘Palestinian National Authority’ (PNA) letterhead,8 provides:

In conformity with Article 12, paragraph 3 of the Statute of the International Criminal Court, the Government of Palestine hereby recognizes the jurisdiction of the Court for the purpose of identifying, prosecuting and judging the authors and accomplices of acts committed on the territory of Palestine since 1 July 2002.

The present article examines the capacity of the Palestinian declaration to provide the ICC with jurisdiction based on a territorial nexus, focusing on three issues regarding the admissibility of the Palestinian declaration under Article 12(3), all of which are linked to the question of statehood. Following a brief review in part 2 of the need to rely on territorial jurisdiction, part 3 addresses the question whether the Palestinian declaration can be admitted9 as that of a state. This article argues that regardless of the normative arguments regarding the existence and recognition of a Palestinian state,10 the ICC Prosecutor may not assume the existence of a Palestinian state because the Palestinians themselves do not make a claim to that effect. Part 4 examines a more nuanced question: whether under a purposive interpretation of Article 12(3), declarations should also be admitted from quasi-states such as the PNA. Part 5 focuses on institutional considerations, examining the consequences of the ICC Prosecutor engaging in questions concerning statehood and recognition. A separate issue, not analyzed here, is the geographical areas which are covered by the declaration, and specifically whether the territory of ‘Palestine’ that is governed by the PNA extends to the Gaza Strip.11

ICC proceedings are only one element in the international community’s endeavor to secure accountability for violations of international law.

While this article concludes that the ICC Prosecutor should not admit the declaration as a basis for jurisdiction, it should be emphasized that ICC proceedings are only one element in the international community’s endeavor to secure accountability for violations of international law, in general and with respect to the Gaza Strip specifically. The ICC is not the only mechanism for accountability, even if it is the most structured and visible one. Moreover, its mandate is by definition limited to individual conduct, and does not extend to violations of the laws of armed conflict that do not amount to criminal acts.12 In the circumstances, accountability could more usefully be pursued through other mechanisms, such as the exercise of universal jurisdiction by individual states, effective incentives for involved parties to enforce the law within their domestic systems, and invocation of state responsibility.

2. Preconditions for ICC Jurisdiction

The ICC may only exercise its jurisdiction over genocide, crimes against humanity and war crimes if the accused is a national of a state which has accepted the jurisdiction of the ICC; if the crime took place on the territory of a state which has accepted ICC jurisdiction; or if the United Nations Security Council refers the situation to the Prosecutor, irrespective of the nationality of the accused or the location of the crime.13 In the circumstances of the Gaza Strip, none of the preconditions for ICC jurisdiction (a territorial or nationality nexus, or a Security Council referral) are met in a manner which offers a straightforward basis for ICC jurisdiction. Indeed, according to media reports, the ICC Prosecutor initially announced that he was unable to open investigations into alleged Israeli violations of the laws of armed conflict during the Gaza Conflict, because the Court lacked jurisdiction.14 The nationality nexus is not directly applicable with respect to Israelis, because Israel is not party to the ICC Statute.15 It may be possible to bring Israeli nationals before the ICC if they are also nationals of other states that are parties to the ICC Statute.16 Nonetheless, reliance on multiple nationalities as the basis for initiating an investigation raises various challenges and difficulties, and the practice of the ICC Prosecutor to date indicates a reluctance to rely on multiple nationality as a nexus in instances involving large contingencies of military and other persons who act on behalf of non-party states.17

Referral of the situation to the Prosecutor by the Security Council acting under Chapter VII of the UN Charter18 is also an unlikely scenario: the United States, despite its gradual warming towards the ICC,19 is likely to veto any attempt to pass the necessary resolution.20 This is probably the background for the Fact Finding Mission’s proposal that the General Assembly have recourse to Resolution 377(V), ‘Uniting for Peace,’21 as a means of initiating criminal prosecutions, even though this approach is unprecedented.

The territorial basis for jurisdiction, which requires acceptance of ICC jurisdiction by the state on the territory in which the alleged crimes were committed, does not provide a simple basis for ICC jurisdiction either.22 The crimes are alleged to have been committed in the Gaza Strip, which is not under the sovereignty of any state presently party to the ICC Statute. The Palestinian declaration is an attempt to provide the territorial nexus necessary for jurisdiction. It invokes ICC Statute Article 12(3), which provides that where ‘the acceptance of a State which is not a Party’ to the Statute is required for the ICC to have jurisdiction over a crime, that state may accept the exercise of jurisdiction by the Court by lodging a declaration with the ICC Registrar. The purported acceptance of jurisdiction is not limited to acts committed during the December 2008–January 2009 conflict, but extends retroactively to 1 July 2002, the date of entry into force of the ICC Statute and the earliest date regarding which the Court may exercise jurisdiction. The ICC Prosecutor responded to the Palestinian declaration by announcing that he ‘will carefully examine all relevant issues related to the jurisdiction of the Court, including whether the declaration by the Palestinian National Authority accepting the exercise of jurisdiction by the ICC meets statutory requirements; whether the alleged crimes fall within the category of crimes defined in the Statute, and whether there are national proceedings in relation to those crimes’.23

Against this background, the potential significance of the Palestinian declaration under Article 12(3) becomes apparent. Yet since Article 12(3) speaks of a declaration by a state not party to the ICC Statute, prima facie, the declaration is inadmissible. The following analysis considers three aspects of this question.

3. Admission of the Palestinian Declaration as that of a Full-Fledged State

Neither the PLO nor the PNA claims that a state of Palestine already exists. Instead, they continue to demand the establishment of a sovereign and independent state.

Under a straightforward, literal interpretation of Article 12(3), for the Palestinian declaration to be admitted under Article 12(3) it must be regarded as having been issued by a state. Views on whether a state of Palestine can or should be recognized vary,24 and this article does not purport to arbitrate among them. In fact, it is submitted that the debate is largely beside the point. For present purposes, it may be assumed that the PNA exercises sufficiently effective control in the West Bank to fulfil the factual criteria of statehood, and that the right of the Palestinian people to self-determination can cure the weakness of such control. However, statehood is a claim of right and not only of fact.25 Even if the PNA has in fact acquired extensive control over the population in the territory, effective control and recognition cannot consolidate the statehood of an entity which does not claim such status.26 Yet neither the PLO nor the PNA27 claims that a state of Palestine already exists. Instead, they continue to demand the establishment of a sovereign and independent state.28 The agreements that the PLO has signed with Israel29 and the rhetoric of the PNA all indicate that independence is regarded as a goal rather than a status already achieved.30 This is more than a political stance; the Palestinians argued before the ICJ that as a matter of law, a state of Palestine does not yet exist, expressly stating that ‘Israel and Palestine are not two States Members of the United Nations’, that ‘[t]he people of Palestine have an unfulfilled right to self-determination’,31 and that the court is ‘not asked to determine the boundaries of a future Palestinian State’.32 That the Palestinians demand to exercise the right to self-determination is undeniable — but the demand itself towards an addressee apparently capable of enabling that exercise indicates that the Palestinians themselves are at least ambivalent as to whether they exercise the requisite territorial control, and at any rate do not yet wish to be regarded as independent. Ironically, pronouncing that a Palestinian state exists despite the absence of a Palestinian claim to this effect may even amount to a violation of that Palestinian right to self-determination, because it would impose a political status on the Palestinian people which they have not yet asserted.

One might argue that the deposit of the declaration is itself an implicit claim of statehood. However, claims to statehood cannot be inferred from statements or actions short of explicit declaration.33 Moreover, the ambiguity of the Palestinian declaration precludes the conclusion that it implicitly asserts statehood. In particular, the term ‘PNA’ invokes a governmental apparatus which was established by an Israeli-PLO agreement, premised on the non-sovereignty of the PNA. Since this term acknowledges a non-state status, its use is irreconcilable with a claim to statehood.34 In the immediate term, an assertion of Palestinian statehood would also undermine the claim that Israel remains an occupying power in the Gaza Strip even after its disengagement.35 Statehood and occupation are of course not mutually exclusive, in the sense that occupation of a state does not extinguish sovereignty despite the loss of effective control by the ousted sovereign.36 But since the West Bank and Gaza Strip were not under independent sovereignty prior to their occupation by Israel, the claim can be, at best, that independence emerged from under occupation, namely that non-sovereign territory has become independent in the face of occupation.37 Since both occupation and independence assume effective control by opposing parties, independence implies that the occupant has been repelled, even if only partially.38 Thus, in order to claim independence, the Palestinians must indicate some area over which they already exercise effective control to the exclusion of Israel. The likely candidate territory is the Gaza Strip, where Israel has far less control than in the West Bank. While the question whether the territory is occupied depends on objective facts rather than on the parties’ claims, the Palestinians cannot argue that the territory is both independent and occupied at the same time. Indeed, some of the Palestinian arguments on why the declaration should be accepted as that of a state reportedly rely on Israel’s denial of its status as an occupying power.39 As noted earlier, a claim of statehood would also raise the question of the relationship between the West Bank and Gaza Strip, given that each area is controlled by a different authority.40

In conclusion, the main obstacle to regarding the Palestinian declaration of 21 January as a valid acceptance of ICC jurisdiction under ICC Statute Article 12(3) is the absence of a Palestinian claim of statehood, which is a prerequisite for the exercise of the power under Article 12(3).

4. Admission under Article 12(3) of a Declaration by a Quasi-State

Absence of a full-fledged state does not mean that the PNA has no international status.41 For many years it has been referred to as in statu nascendi,42 although the rights that attach to this status remain controversial. Against this background, the question arises whether the term ‘state’ in Article 12(3) should be interpreted more widely, so as to encompass quasi-state, namely territorial entities in which a governmental authority exercises control but which fall short of full-fledged statehood.

It has been suggested by advocates of admitting the Palestinian declaration that expanding the interpretation of the term ‘state’ in Article 12(3) is the only way to prevent a vacuum in criminal accountability insofar as the territory of the PNA is concerned.43 The entire framework of international criminal law, and of the ICC mechanism within it, is premised on the interest in ending impunity being an international, communal one, rather than that of individual states.44 Arguably, this communal interest should not be constrained, in the context of the ICC Statute and more specifically of Article 12(3), by limitations of the traditional meaning of statehood, which makes status subject to political, subjective stances on assertion and recognition of statehood. This is particularly true with respect to the Palestinian territories, which are unique in being neither claimed by any existing state nor recognized as belonging to one. Admission of an Article 12(3) declaration by the PNA would therefore not encroach on any state’s sovereignty, and not jeopardise the basic tenets of the ICC mechanism.

But the Palestinian territories are not in a legal jurisdictional vacuum. The ICC provides a mechanism that can cover them, namely a referral by the Security Council. If the Council refuses to exercise its power, it is acting on the political prerogative which the ICC Statute drafters knew it to possess and have agreed to tolerate.45 Moreover, the inapplicability of a statute of limitations to international crimes is designed to overcome political impediments to accountability in the short term. This leaves open, at least in theory, the possibility of a future referral by the Security Council. The fact that at a specific moment in time, with respect to a specific situation, none of the three bases for jurisdiction (territory, nationality or a Security Council referral) has successfully led to acquisition of jurisdiction does not mean that the rules on jurisdiction are inadequate, but merely demonstrates that the ICC is not omnipotent. It is worth noting that the status of the Palestinian territories as non-sovereign territory under control of a non-state entity never arose during the drafting of the Statute. Arguably, given the prominence of the territories in other aspects of the negotiations, the absence of territorial nexus can hardly be considered an oversight; rather, it seems that the drafters were content to accept the consequences of the limits of that nexus.46

One might argue that given the political constraints on Security Council action, reliance on its powers to reject innovative alternatives is overly formalistic. But the ICC is nothing but a formal mechanism: it is aimed to fill an institutional gap so as to enable implementation of international legal norms that for the most part already exist under substantive international law. Its structure may have no other advantage over alternative ones other than having been agreed upon. Since it is this agreement that gives it legitimacy and enables it to fill the prior institutional gap, there is merit in a strict adherence to it.

To conclude, while it is not unthinkable that the meaning of the term ‘state’ for the purposes of the Article 12(3) be different than its meaning for the purposes of customary international law,47 the ICC Statute should be interpreted in good faith in accordance with the ordinary meaning to be given to its terms in their context and in the light of its object and purpose.48 The object and purpose of the ICC Statute do not call for any digression from this ordinary meaning.

Re-interpreting the term in Article 12(3) may have a destabilizing effect on the interpretation of other provisions of the Statute applicable to states not parties, such as those concerning the principle of complementarity and challenges to jurisdiction.49 There may also be a spillover effect to other terms used in the Statute. For example, it might be suggested that the term ‘national’ in Article 12(2)(b) includes permanent residents.50 In the context of the laws of armed conflict the ICTY has already expanded the term, ruling that the term ‘national’ in Article 4 of the Fourth Geneva Convention should be interpreted as relating to ethnicity rather than to formal bonds and purely legal relations.51 However, that interpretation was given ‘within the context of the changing nature of the armed conflicts since 1945, and in particular of the development of conflicts based on ethnic or religious grounds’.52 If the immense legal and political changes that the world has undergone during over half a century permit a purposive interpretation of the Fourth Geneva Convention, the same cannot yet be said for the ICC Statute, adopted only a decade ago. Moreover, unlike the Fourth Geneva Convention, the ICC Statute is a criminal code which is subject to principles of interpretation applicable within penal law. These call for a strict reading of terms, less amenable to policy considerations. An exceptionally liberal interpretation of the Statute that departs from this interpretative standard also risks deterring other states still not party to the ICC Statute from joining, making a novel interpretation of the term ‘state’ untimely as well as legally dubious.53

The governmental authority lodging the declaration is not the one exercising criminal jurisdiction.

Doctrinally, to apply Article 12(3) to non-state entities it is essential that the entity in question be capable of fulfilling the functions envisaged by the Statute, in this case the delegation of criminal jurisdiction.54 The requisites for this are possession of jurisdiction over criminal matters (a requisite also underlying the principle of complementarity) and delegation of that jurisdiction to the ICC.55 Different opinions have been proffered as to whether the PNA enjoys criminal jurisdiction in the territories under its control, generally and specifically over Israelis. These usually focus on the regulation of the matter under the Israeli-PLO Interim Agreement and on the validity of this Agreement.56 However, whether the Palestinians have criminal jurisdiction over the Gaza Strip for the purpose of delegating that jurisdiction to the ICC is not a matter which is governed by the Interim Agreement, which is a bilateral instrument, and its effect as against third parties is limited. Rather, the criminal jurisdiction of the PNA for the purpose of its delegation to a third party depends on the objective status of the territory that is applicable erga omnes. Michael Kearny and Al Haq offer a more nuanced approach, claiming that regardless of the PNA’s status, the notion of universal jurisdiction provides it with the jurisdiction and capacity to delegate to the ICC.57 However, since universal jurisdiction for grave breaches of international humanitarian law is incumbent upon states, the intractable question of status cannot be circumvented. If Israel continues to be held an occupying power, it must be regarded as continuing to have criminal jurisdiction.58 If, on the other hand, the occupation of the Gaza Strip has ended by virtue of Israel’s disengagement from it in 2005, jurisdiction may well have passed into the hands of the Palestinians.59 Yet since June 2007, it is also not the PNA who is in effective control over the Gaza Strip but Hamas. In short, the governmental authority lodging the declaration is not the one exercising criminal jurisdiction.

5. Institutional Considerations

Given the indeterminate status of the PNA, any pursuit of its declaration by the ICC Prosecutor would constitute at least an implicit recognition of the international status of that entity, whether as a full-fledged state or as a state for the purpose of Article 12(3). In addition to the substantive objections to admission of the Palestinian declaration under Article 12(3), there are also institutional considerations militating against such admission.

Although recognition of statehood is commonly regarded as a declaratory act and not as a constitutive requisite for statehood,60 in borderline cases such as that of the PNA it may constitute a step toward consolidating an indeterminate general legal status.61 For this reason, a determination by a legal body such as the ICC (the prosecutor and, at a later stage, the Court) that a state of Palestine exists (either generally or for the purpose of Article 12(3)) would carry significant weight.

Recognition of an entity as a state is a political act, traditionally within the prerogative of states. There is nothing in international law precluding an international actor such as the ICC Prosecutor or Court from extending recognition to a state.62 Ordinarily, however, international organisations and their organs do not recognise states. Their treatment of entities as states is only a consequence of the prior recognition by member states of those entities’ statehood. Even individual states are reluctant to extend recognition of statehood when acting in their capacity as institutional organs.63 It is noteworthy that the depositary in that event, the Swiss Federal Council, was undoubtedly capable and authorized to take a decision as to its own recognition of the applicant.64

Thus, if the Prosecutor, or later the Pre-Trial Chamber, determine that the Palestinian declaration fulfils the requirements of Article 12(3), they would be assuming an almost unprecedented competence, which incurs onto the political sphere which is the traditional prerogative of states. The converse, i.e. a determination that the PNA has not established itself universally as a state would not exceed the ordinary powers of a non-state actor.65

Importantly, unlike organs of other international institutional organs such as treaty depositaries, the ICC Prosecutor cannot refer the decision elsewhere. He alone is mandated with the power and responsibility to make the preliminary decision whether to initiate an investigation.66 Although the Court will always make the final determination as to jurisdiction, even the Prosecutor’s initial decision to investigate could be deemed an act of recognition, thereby politicizing the functions of his office. The Pre-Trial Chamber’s will have automatic power of judicial review over the Prosecutor’s decision if the Prosecutor decides that the ICC has and should exercise jurisdiction. If the Prosecutor decides in the negative, the review by the Pre-Trial Chamber is dependent on a request by the state making the referral.67 The question would then arise directly before the Pre-Trial chamber whether ‘Palestine’ constitutes a state capable of making a request for a review by the Pre-Trial chamber.

With respect to recognition of statehood, the Assembly of States Parties (ASP) is a possible forum from which the ICC Prosecutor may wish to take guidance. In view of the administrative and managerial character of the specific tasks allocated to the ASP,68 it is doubtful whether the drafters intended for this power to extend to political questions that have a direct effect on the jurisdiction of the Court, which has been so rigorously negotiated;69 yet the ASP may engage in any function necessary or essential for the Court and consistent with the Statute.70 The ASP also has authority to settle disputes between states parties as to interpretation or application of the Statute.71 This may include interpretation of the term ‘state’ for the purpose of Article 12(3) or even recognition of statehood. How the ASP’s decisions on such matters will affect the powers of the Prosecutor’s and pre-Trial Chamber remains unclear.

It has been argued that the consequences of admission of the Palestinian declaration as that of a state should not be overstated. The direct effect of such admission would be only to grant jurisdiction to the ICC with respect to a specific situation. It would not determine status for general purposes, nor even for other provisions of the ICC Statute.72 Interestingly, the concept of Palestinian statehood for a limited purpose was recently embraced by no other than Israeli courts, which enquired whether the PNA was recognized (by Israel) as a state for the purpose of state immunity, notwithstanding the clear absence of recognition by Israel of Palestinian statehood in general.73 However, the notion that a determination by the ICC Prosecutor or Court can be isolated and restricted to the specific context of ICC territorial jurisdiction is largely illusory.74 Statehood is for the most part a package deal. Where non-states have been granted rights and obligations which ordinarily attach only to states, this was usually not through ad hoc recognition or limited statehood but through express extensions of those rights and obligations to non-state entities75 or through the gradual expansion of international law to non-state entities.76 There are few and remote precedents of an entity being regarded under international law as a state for some purposes but not for others.77 Thus, it would be naïve to expect that recognition by a legal organ of an international organisation which brings together over half of the worlds’ states would have no repercussions outside the immediate context in which such recognition was made.

A further argument against the Prosecutor undertaking a decision as to the status of the PNA is that this would create a precedent for use of the ICC as a forum from which non-state actors could publicly assert political independence from their parent states. It would be an invitation to aspirant entities of diverse types, such as Kosovo, Taiwan, South Ossetia, Abkhazia, Transdnistria, Somaliland and the Turkish Republic of Northern Cyprus, which have not managed to garner sufficient international support for a status they claim, to try to advance their goals though the ICC. Regardless of the prospects of success, the existence of a new, international forum which recognizes states by reference to non-classical considerations (i.e. criminal jurisdiction and capacity to delegate it to the ICC) invites its abuse. For example, if crimes were committed in Kosovo by nationals of states not parties to the ICC Statute, could the Prosecutor or Court rely on a Kosovar declaration under Article 12(3)?78 Similarly, Polisario, the internationally-recognized representative of the Saharawi people79 and governing body of the aspirant Saharawi Arab Democratic Republic (SADR),80 might lodge a declaration with respect to crimes perpetrated by nationals of non-party states in the territory it controls. Another case in point may be that of Taiwan. China is not a party to the Statute (nor is Taiwan, of course). Could the ICC Prosecutor assert ICC jurisdiction over these crimes on the basis of a Taiwanese declaration under Article 12(3)? In all these cases the answer would depend on whether the Prosecutor and later Court consider the aspirant entities to be states.

Arguably, the ICC can be prevented from becoming the battle ground over status claims if Article 12(3) is interpreted as allowing declarations to be lodged only by non-state entities governing territories over which there is no competing territorial claim. This would bar potential declarations by Kosovo or Taiwan, for example. Should the SADR lodge a declaration, the situation would be more complicated: although Morocco claims Western Sahara as sovereign territory, the ICC Prosecutor would be hard put to take account of this claim, because this may be interpreted as contrary to the findings of the ICJ in its advisory opinion, and consequently to the international commitment to the right to self-determination of the Saharawi people.81 But more generally, as argued above, there is no situation which cannot be addressed by the Security Council and which requires additional means of granting the ICC jurisdiction. Under these circumstances, any permissive criterion would be suspect of reflecting a political desire to apply special criteria to the PNA rather than an objective policy.

In theory, barring Article 12(3) declarations by quasi-states would not prevent issues relating to jurisdiction over disputed territory from arising in the ICC. For example, if an established state (eg Serbia, China or Morocco) asserts that its consent to jurisdiction provides a territorial nexus to territory under the aspirant entity’s administration, the ICC Prosecutor would still have to take a decision on territorial jurisdiction in light the status of the competing claim by the aspirant state. However, in practice such a scenario is not likely to occur. For example, it seems unlikely that China or Morocco would lodge a declaration under Article 12(3) with respect to acts committed in Taiwan or Western Sahara, respectively. This is because non-party states are unlikely to put their territorial status under international legal scrutiny in order to secure accountability of a handful of individuals. If they do lodge such declarations, this will more likely be in furtherance of their own territorial claims. Such use of the Court should not be encouraged any more than declarations by non-state entities. The likelihood that territorial disputes arise through referral by other states is even less likely, simply because states that are not directly involved have so far demonstrated reluctance to engage in referrals, even in territorially-undisputed situations. Finally, a referral by the Security Council obviates the question of territorial nexus. Thus, the potential admission of an Article 12(3) declaration by quasi-states that holds the greatest risk for politicizing the ICC in this context.

In conclusion, a decision by the ICC Prosecutor granting status under the ICC Statute to a non-state entity would be an irregular event in the practice of international organisations, the merit of which would be seriously outweighed by negative costs to both the ICC as an institution and to the discipline of international criminal justice more broadly.

6. Conclusion

The present article considers the possibility of ICC jurisdiction over crimes allegedly committed in the Gaza Strip based on a territorial nexus asserted by the PNA. From the analysis it appears that creating such a nexus has no legal ground and is politically precarious. There is no doubt that the PNA is very close to becoming a state; in fact, the only bar may be the absence of declaration of statehood on its part, as the requisites of effectiveness may have already been fulfilled at least to a minimal level that together with the right to self-determination can create a presumption of statehood. However, at present the Palestinian leadership does not assert statehood. Thus, it would be premature for the ICC Prosecutor or Court to recognise the Palestinian declaration as that of a state, even for the limited purpose of Article 12(3). Interpreting Article 12(3) more widely to include entities effectively governing non-sovereign territory also seems unwarranted, as such interpretation flies in the face of the ICC Statute’s wording and the intention of its drafters. Any involvement in issues of recognition risks exposing the Prosecutor and the Court to accusations of politicisation and subjectivity.

The ICC’s goal of ending impunity is channelled through a state-centred mechanism. Despite being an international tribunal, the ICC is more restricted than states since it does not have original, universal jurisdiction. Undoubtedly, the ICC mechanism sometimes leaves justice hostage to political forces. In particular, involvement in unresolved political conflicts may entangle it in questions exceeding its mandate as envisaged by its founders.82 Extricating the ICC from these entanglements should be done through careful, measured tugs at the limits of the delicate balance achieved between accountability and sovereignty. These limitations of the ICC have been candidly admitted by the former president of the ASP and head of the Jordanian delegation to the ASP, His Royal Highness Prince Zeid Raad Al-Hussein, who noted, with regard to the Palestinian declaration, that ‘…whenever we believe injustice has intruded upon our lives in the Middle East we scream for the International Criminal Court and yet many of us never seem to read the Statute properly, and in particular articles 12 and 13. If more of us read those articles, we would be more understanding of how the Court — rightly or wrongly — was designed to operate’.83

The ICC is not and should not be regarded as a panacea. Its limits should not be disregarded, as this would jeopardise the Court’s legitimacy and effectiveness. Yet accountability should be ensured. It should be sought in other international institutions. Some address themselves directly to individual criminal responsibility, such as the principle of universal jurisdiction. Others address themselves to the responsibility of states, such as the principles of state responsibility for wrongful acts under international law. Others still may exist that concern the international responsibility of states, e,g, under international human rights law, to ensure individual accountability under its domestic law. These and other institutions may be preferable to resorting to the ICC, for example where they cast a wider net of accountability, or provide an effective incentive for policy changes that further peace, security and wellbeing.84 They should not be discarded in favour of the ICC, simply because it is the newest addition to the architecture of international adjudication.

Endnotes — (click the footnote reference number, or ↩ symbol, to return to location in text).

  1. .

    This article is an abridged and slightly revised version of an article published in 8 Journal of International Criminal Law (2010) 3–27. I am grateful to OUP for permission to use the article.

    Research for this article was carried out during my year as 2008–2009 Visiting Research Fellow at the Hebrew University’s Minerva Center for Human Rights, made possible by the Minerva Center’s Vidal Angel Postdoctoral Program for Research against Hate and Bigotry. e-mail: yael.ronen@cantab.net. I am grateful to James Crawford, Jonathan Heuberger, Claus Kress, Gilad Noam, Robbie Sabel, Bill Schabas, Yuval Shany, James Stewart, Daniel Taub, Nicholas Tsagourias and the anonymous reviewers for their comments on earlier drafts. Usual disclaimers apply.

  2. 1.

    E.g. UN Doc. S/PV.6061, 6 January 2009, and UN Doc. S/PV.6061 (Resumption 1), 7 January 2009; Statement by 31 international lawyers, Israel’s bombardment of Gaza is not self-defence — it’s a war crime, The Sunday Times, 11 January 2009, available online; Israel Ministry of Foreign Affairs, The Operation in Gaza—Factual and Legal Aspects, July 2009, available online, § 52–76; Human Rights Watch, Q&A: Accountability for Violations of International Humanitarian Law in Gaza, 6 February 2009, available online, last visited 15 December 2009; Amnesty International, Growing calls for investigations and accountability in Gaza conflict, 14 January 2009, available online, last visited 15 December 2009.

  3. 2.

    General Assembly Demands Full Respect For Security Council Resolution, 1860, UN Doc. GA/10809/Rev. 1, 16 January 2009 (Statement of Bolivia), available online, last visited 15 December 2009; Human Rights Watch, Israel/Gaza: International Investigation Essential, 27 January 2009, available online, last visited 15 December 2009; sixteen international lawyers in a letter entitled Find the truth about the Gaza war, 16 March 2009, available online, last visited 15 December 2009; Amnesty international, Evidence of misuse of US-weapons reinforces need for arms embargo, 23 February 2009, available online, last visited 15 December 2009; ACRI and Organizations: Investigate Israel’s Attacks on Civilians, 20 January 2009, available online, last visited 15 December 2009; The ICC Prosecutor has received over 210 appeals from Palestinians and NGOs to investigate the Israeli-Palestinian conflict, Interview with Luis Moreno-Ocampo, ICC Prosecutor, on the Riz Khan show, 19 March 2009, Aljazeera Television, available online, last visited 15 December 2009. These include complaints predating the Gaza Conflict, RNW International Justice, ICC starts analysis of Gaza war crimes allegations, 3 February 2009, available online, last visited 15 December 2009.

  4. 3.

    Human Rights Council Resolution S-9/1 on the Grave Violations of Human Rights in the Occupied Palestinian Territory, Particularly due to the Recent Israeli Military Attacks Against the Occupied Gaza Strip, UN Doc. A/HRC/S-9/L.1, 12 January 2009.

  5. 4.

    Report of the United Nations Fact Finding Mission on the Gaza Conflict, UN Doc. A/HRC/12/48, 25 September 2009 [hereinafter cited as Goldstone Report], §§ 1886, 1891, 1921, 1950.

  6. 5.

    Goldstone Report, supra note 4, § 1969.

  7. 6.

    Human Rights Resolution S-12/1 The human rights situation in the Occupied Palestinian Territory, including East Jerusalem, UN Doc. A/HRC/RES/S-12/1 B, 16 October 2009, § 3.

  8. 7.

    Declaration recognizing the Jurisdiction of the International Criminal Court, available online, last visited 15 December 2009.

  9. 8.

    The Independent Fact Finding Committee appointed by the League of Arab States [hereinafter cited as Arab League Committee] criticized the ICC for exceeding its authority by “changing the Government of Palestine into the PNA” in its response to the declaration. Given the fact that the declaration was submitted on PNA letter head, this assertion is puzzling. Report of the Independent Fact Finding Committee on Gaza: No Safe Place, Presented to the League of Arab States, 30 April 2009, § 594, available online, last visited 15 December 2009.

  10. 9.

    The present article uses the term “admission” to distinguish admissibility of the declaration from acceptance of jurisdiction within the declaration.

  11. 10.

    Compare with J. Quigley, The Palestine Declaration to the International Criminal Court: The Statehood Issue, 35 Rutgers L. Rev. (2009) 1 , at 9, who considers the question of statehood in the same context as the present article but only relates to that context in the penultimate paragraph of the conclusion.

  12. 11.

    On the one hand there is an international commitment to the unity of the two areas as a single political entity. On the other hand, since June 2007, when Hamas took over control of the Gaza Strip, the PNA has lost control over that area. One may argue that a state of Palestine asserted by the PNA must be limited to the West Bank; alternatively, if the Gaza Strip constitutes a sovereign state, its government is not the PNA but the Hamas, which has not lodged any declaration with the ICC. Resolution of this issue requires an analysis of the relationship between the PNA and Hamas, which is outside the scope of this article.

  13. 12.

    See OTP response to communications received concerning Iraq, available online, last visited 15 December 2009.

  14. 13.

    Acceptance can be effected by accession to the ICC Statute or by an ad hoc declaration, Arts 13, 12(3) ICCSt.

  15. 14.

    RNW International Justice, supra note 2; M. Simons, Palestinians Press for War Crimes Inquiry on Gaza, New York Times, 11 February 2009, at A13, available online, last visited 15 December 2009.

  16. 15.

    On Israel’s position towards the ICC see Shany, Yuval, The Entry into force of the Rome Statute: What are its implications for the State of Israel, 15 Hamishpat [in Hebrew] (2003) 28; R.J. Goldstone, Israel and the International Criminal Court, 15 Hamishpat [in Hebrew] (2003) 12; D.A. Blumenthal, The Politics of Justice: Why Israel Signed the International Criminal Court Statute and What the Signature Means, 30 Georgia Journal of International and Comparative Law (2002) 593.

  17. 16.

    According to the press, the ICC Prosecutor ‘is considering’ taking such a step against an Israeli reserve officer who is also a national of South Africa. D. Ephron, ICC Prosecutor May Charge Israeli With War Crimes, Newsweek, 21 September 2009, available online, last visited 15 December 2009.

  18. 17.

    In February 2006 the ICC Prosecutor made public his reasons not to initiate investigations on the basis of the nationality nexus with respect to crimes allegedly committed on the territories of Iraq and Venezuela, both non-parties to the ICC Statute. OTP response to communications received concerning Iraq, supra note 12, and OTP response to communications received concerning Venezuela, available online, last visited 15 December 2009.

  19. 18.

    Art. 13(b) ICCSt.

  20. 19.

    For a review of US policy towards the ICC see ASIL Independent Task Force, U.S. Policy Toward the International Criminal Court: Furthering Positive Engagement, March 2009, available online, at 5–17, last visited 15 December 2009.

  21. 20.

    JTA (Washington), White House: Official ‘misspoke’ on Goldstone report, 23 September 2009, available online, last visited 15 December 2009. Previously, the US did not oppose the referral of the situation in Sudan to the ICC (S.C. Res. 1593 (2005), 31 March 2005). At the same time, the statement of the US Permanent Representative to the UN, supra note 1) does not offer much prospect for the US acquiescing in a referral of the Gaza Conflict to the ICC.

  22. 21.

    Goldstone Report, supra note 4, § 1971. The Arab League Committee also suggested that if the Security Council fails to refer the situation to the ICC, the League of Arab States should request the General Assembly to “endorse Palestine’s declaration”, in a meeting constituted in terms of the Uniting for Peace Resolution, GA Res. 377(V)A), 3 November 1950.

  23. 22.

    Neither Jordan nor Egypt, both of which had previous territorial links to the Palestinian territories, can be considered the territorial states. Jordan, an ICC state party, purported to annex the West Bank in 1950. Only two states have ever recognized this annexation. In 1988 Jordan renounced its legal and administrative claims to the West Bank (Jordan: Statement concerning Disengagement from the West Bank and Palestinian Self-Determination, 31 July 1998, 28 International Legal Materials (ILM) (1988) 1637). Reliance on a territorial nexus to Jordan is therefore weak because it is based on a disputed claim that has voluntarily been terminated twenty one years ago. It may also be contrary to the peremptory obligation to respect the right of the Palestinians to self-determination within (at least) the West Bank and Gaza Strip (East Timor Case (Portugal v Indonesia), International Court of Justice, 30 June 1995, ICJ Reports (1995) 90, at 120, § 29; ILC Draft Articles on State Responsibility for Internationally Wrongful Acts, UN Doc. A/56/19 (2001), 113, commentary to Art. 40, § 5). Finally, the Jordan territorial link does not cover the Gaza Strip. The latter had been from 1948 until 1967 under Egyptian military rule. Egypt (not an ICC state party) held the Gaza Strip under occupation from 1948 to 1967, never claiming sovereignty over it. In an era of post-conflict State-building occupation, the notion that an occupying power as representing the interests of the population, including through delegation of jurisdiction to the ICC, is not unthinkable. But a leap of logic would be required to regard Egypt as the relevant occupying power rather than Israel, which is either the current occupant or the most recent occupant.

  24. 23.

    ICC Office of the Prosecutor Press Release, Visit of the Minister of Justice of the Palestinian National Authority, Mr. Ali Khashan, to the ICC (22 January 2009), 6 February 2009, available online, last visited 15 December 2009. The ICC Prosecutor has announced that if he finds that he can investigate the Gaza situation on the basis of a territorial nexus, he will examine the conduct of both sides. Interview with Luis Moreno-Ocampo, supra note 2. This statement is in line with Rule 44(2) ICC RPE, which is intended to prevent an interpretation of Art. 12(3) ICCSt. which allows a one-sided declaration aimed at the adversary while sheltering the declaring state. S.A. Williams and W.A. Schabas, Article 12, in O. Triffterer (ed.) Commentary on the Rome Status of the International Criminal Court (2nd edn., C.H. Beck, Hart, Nomos: München, 2008) 547, at 559. In the circumstances, the problem does not seem to arise as the declaration is drafted in a general manner. For doubt as to the effectiveness of rule 44(2) see J. Goldsmith, The Self-Defeating International Criminal Court, 70 U. Chi. L. Rev. (2003) 89, at 92 ft 11.

  25. 24.

    E.g. A. Boyle, Creation of the State of Palestine, 1 EJIL (1990) 301, and response by J. Crawford, The Creation of the State of Palestine: Too Much Too Soon?, 1 EJIL (1990) 307; J. Quigley, Competing Claims to the Territory of Historical Palestine, 59 Guild Practitioner (2002) 76. The same arguments are essentially reproduced in Quigley, supra note 10; J. L. Prince, The International Legal Implications of the November 1988 Palestinian Declaration of Statehood, 25 Stanford Journal of International Law (1989) 681; S. R. Silverberg, Diplomatic Recognition of States in Statu Nascendi: The Case of Palestine, 6 Tulsa Journal of Comparative and International Law (1998–1999) 21; K. M. McKinney, The Legal Effect of the Israeli-PLO Declaration of Principles Toward Statehood for Palestine, 18 Seattle University Law Review (1994–1995) 93; P.J.I.M. de Waart, Self-Determination in Perspective: Palestinian Claims to Statehood and the Relativity of the Right to Self-Determination, 8 Palestinian Yearbook of International Law (1994–1995); T. Becker, Self-Determination in Perspective: Palestinian Claims to Statehood and the Relativity of the Right to Self-Determination, 32 Israel Law Review (1998) 301; Y.Z. Blum, A. Gerson, J. Quigley and Y. Ronen. Nakhleh in Self Determination: The Case of Palestine, 82 Proceedings of the American Society of International Law (1988) 335. M.G. Kohen, Introduction, in M.G. Kohen (ed.) Secession, International Law Perspectives, (Cambridge: Cambridge University Press, 2006) 1, at 13; Goldsmith, supra note 23, at 94, ft 18, considers the question briefly specifically in the context of Art. 12(3) ICCSt. To the issues raised traditionally, one may add certain challenges that have emerged in the last few years, such as the legal and practical consequences of Israel’s disengagement from the Gaza Strip, and the implications of the political rift between the West Bank and the Gaza Strip.

  26. 25.

    J. Crawford, The Creation of States in International Law (2nd edn. Oxford: Oxford University Press, 2006), at 211.

  27. 26.

    D.P. O’Connell, The Status of Formosa and the Chinese Recognition Problem, 50 Am. J. Int’l L. (1956) 405, at 415; Restatement (Third) of the Foreign Relations Law of the United State, Section 201. Cf. Crawford, supra note 25, at 211 and V. Lowe, International Law (Oxford: Oxford University Press, 2007), at 165, who see the absence of an unequivocal claim of statehood as the only bar to recognition of Taiwan as an independent state. Roth criticizes this approach both in principle and on factual grounds, indicating certain equivocal statements and practice on the part of the Taiwanese leadership. B.R Roth, The Entity that Dare not Speak its Name: Unrecognized Taiwan as a Right-Bearer in the International Legal Order, 4 East Asia Law Review (2009) 91.

  28. 27.

    The term ‘PNA’ is not mentioned in any of the Israeli-PLO agreements. It is the term used by the Palestinians to indicate the collective of institutions which, as acknowledged by the Palestinians, were established in the framework of the agreements. Website of the Permanent Observer Mission of Palestine to the UN, available online, last visited 15 December 2009. The relationship between the PNA and the PLO is a question which exceeds the scope of this article.

  29. 28.

    E.g. Palestine Liberation Organization, Negotiations Affairs Division, 2009 Negotiations Primer, at 12, available online, archived, (‘The PLO’s primary goals in engaging in direct negotiations with Israel are…fulfilment of the Palestinian right to self-determination through the establishment of an independent and sovereign Palestinian state in the West Bank and Gaza Strip with East Jerusalem as its capital…’); A/63/PV.57, 24 November 2008, at 9, 11 (“Moreover, the Palestinian people and their leadership remain convinced that… the international community will ultimately fulfil its responsibilities by upholding international law and the Charter of the United Nations so as to achieve a peaceful settlement that will give our people the freedom for which they have waited so long and allow them to take their rightful place among the nations of the world…” and “We also call for their help in realizing the Palestinian people’s inalienable rights, including their right to self-determination and to their independent State of Palestine…”).

  30. 29.

    Declaration of Principles on Interim Self-Government Arrangements, 13 September 1993, 32 ILM 1525 (1993); Agreement on the Gaza Strip and the Jericho Area, 4 May 1994, 33 ILM (1994) 622. Israeli-Palestinian Interim Agreement on the West Bank and the Gaza Strip, signed 28 September 1995, 36 ILM 557 (1997) [hereinafter cited as Interim Agreement]; Dajani, supra note 56, at 90 notes that the Declaration of Principles and Interim Agreement elicited the support of the Palestinian population only insofar as they were transitional.

  31. 30.

    Quigley, supra note 10, at 7 argues that Israel’s demand in 1993 that the PLO recognise it implied Israel’s recognition of a Palestinian state, since only states may recognise others. Since an existing state does not require recognition from a new state (although Quigley does suggest (at 8–9) that post-1988 Palestine is identical to post-Ottoman Palestine, essentially claiming that the mandate territory constituted a state and disregarding events since 1948), Israel’s demand had purely political objectives. It therefore makes no sense to attach any implicit legal significance to it. Israel’s demand in 1993 was political in the same sense that today it demands that Hamas recognise its right to exist, without in any way implying that it is the government of a state in the Gaza Strip.

  32. 31.

    Both statements in Legal Consequences of the Construction of a Wall (Advisory Opinion), Oral Proceedings CR 2004/1, 23 February 2004, § 22.

  33. 32.

    Id. at § 33 (emphasis added).

  34. 33.

    Crawford, supra note 25, at 211. As demonstrated by the Taiwanese applications for participation in the work of the UN, even a request to perform a function reserved for states can be drafted sufficiently vaguely so as to avoid a claim of statehood or even that the request itself implies a claim of statehood. Mainland Affairs Council, Position Paper Regarding the Referendum on Joining the United Nations Under the Name of Taiwan, 7 September 2007, no longer available online last visited 15 December 2009.

  35. 34.

    The mention of the PNA alongside the “Government of Palestine” may have been intended to enable the ICC Prosecutor to interpret the declaration as claiming statehood without being explicit about it. Alternatively, it may serve to emphasise that the PNA is the only legitimate executive arm of government for all of Palestine, and avoids any charge of illegitimacy that may be claimed if it is attributed in any way to Hamas.

  36. 35.

    Report of the Special Rapporteur on the Situation of Human Rights in the Palestinian Territories Occupied Since 1967, UN Doc A/62/275, 17 August 2007, §§ 9–10; Goldstone Report, supra note 4, § 279; A/HRC/RES/S-12/1 C preambular § 5 (21 October 2009); S. Bashi & K. Mann, Disengaged Occupiers: The Legal Status of Gaza, Gisha, The Legal Center for Freedom of Movement, January 2007; Report of the Arab League Committee, supra note 8, § 15; I. Scobbie, An Intimate Disengagement: Israel’s withdrawal from Gaza, the Law of Occupation and of Self-Determination, 11 Yearbook of Islamic and Middle Eastern Law (2004–2005) 3. The official Palestinian position is unclear. PNA President Mahmoud Abbas referred in his speeches in the UN exclusively to “siege” and “blockade” over the Gaza Strip, UN Doc A/63/PV.11, 26 September 2008, at 38, S/PV.6061, 6 January 2009, at 5. But see also statements of Palestine referring to the Gaza Strip as occupied territory and to Israel as the occupying power in it in the context of the 2008–2009 offensive, e.g. S/PV.6201, 14 October 2009, at 6, S/PV.6216 (Resumption 1), 11 November 2009, at 20, 21.

  37. 36.

    Crawford, supra note 25, at 73.

  38. 37.

    For present purposes it does not matter whether this took place in 1988 or through the implementation of the Interim Agreement, supra note 29.

  39. 38.

    E.g. the case of Guinea-Bissau, N. MacQueen, Belated Decolonization and UN Politics against the Backdrop of the Cold War: Portugal, Britain, and Guinea-Bissau’s Proclamation of Independence, 1973–1974, 8 Journal of Cold War Studies (2006) 29.

  40. 39.

    C. Philp & J. Hider, Prosecutor looks at ways to put Israeli officers on trial for Gaza “war crimes, Times Online, 2 February 2009, available online, last visited 15 December 2009.

  41. 40.

    Supra note 11.

  42. 41.

    Crawford, supra note 25, at 219. This is by no means a novel notion: R. Roxburgh (ed.), L. Oppenheim, International Law: a Treatise Vol. I (3rd edn, London, New York: Longman, Green and Co., 1921), at 128, 133.

  43. 42.

    E.g. T. Giegrich, The Palestinian Autonomy and International Human Rights Law: Perspectives on an Ongoing Process of Nation-Building, in A. Shapira & M. Tabory (eds), New Political Entities in Public and Private International Law, (The Hague: Kluwer Law International, 1999) 183, at 195.

  44. 43.

    Errol Mendes, Statehood and Palestine for the purposes of Article 12(3) of the ICC Statute: a contrary perspective, 30 March 2010, at 47, available online.

  45. 44.

    Preambular § 5 ICCSt.

  46. 45.

    M. Bergsmo & J. Pejić, Article 16, in Trifterrer, supra note 38, 595, at 598. I differ on this point from Prof. Yuval Shany, who considers the limitations of the Security Council as justifying a functional interpretation of Article 12(3). Shany, Yuval, In Defence of Functional Interpretation of Article 12(3) of the Rome Statute, A Response to Yaël Ronen, 8 J. Int’l Crim. Just. (advance publication), at 9.

  47. 46.

    Michael Kearny offers an explanation for the Palestinian silence at the time, online.

  48. 47.

    F.L. Kirgis Jr., Admission of “Palestine” as a Member of a Specialized Agency and Withholding the Payment of Assessments in Response, 84 Am. J. Int’l L. (1990) 218, at 220. Kirgis’ example, however, is unsatisfactory: he notes divergence of meaning between domestic US law and international law.

  49. 48.

    Vienna Convention on the Law of Treaties (adopted May 23, 1969, entered into force 27 January 1980) 1155 UNTS 331 [hereinafter cited as VCLT], Art. 31.

  50. 49.

    Arts 17, 19 ICCSt. It would probably be unrealistic to suggest that a question on the meaning of “state” would arise with respect to the right to accede to the Statute ICC (Art. 125(3) ICCSt.).

  51. 50.

    Indeed, there is a trend of increasing assimilation of permanent residents to nationals in terms of the State’s human rights obligations towards them. At present this trend seems limited to domestic implementation. States still distinguish between nationals and permanent residents for purposes of international protection. One may argue in favor of such assimilation: permanent residence is a sufficiently-stable relationship with a State so as to justify the imposition on the individual of certain obligations without putting the individual at an unexpected detriment. On the other hand, interpreting the Statute in this way would be clearly contrary not only to the ordinary meaning of the term but also to the intention of the drafters (VCLT, supra note 48, Art. 32).

  52. 51.

    Appeals Judgment, Tadić (IT-94-1), Appeals Chamber, 15 July 1999 § 168; Appeals Judgment, Delalić et al. (IT-96-21), Appeals Chamber, 20 February 2001, §§ 56–73.

  53. 52.

    Id.

  54. 53.

    A matter which is no longer an issue with respect to the Geneva Conventions, in which membership is universal.

  55. 54.

    Kirgis, supra note 47, at 221.

  56. 55.

    H.-P. Kaul, Preconditions to the Exercise of Jurisdiction in A. Cassese, P. Gaeta & J.R.W.D. Jones (eds) The Rome Statute of the International Criminal Court; a Commentary Vol. I (oxford: Oxford University Press, 2002) 583, at 607–610, D. Akande, The Jurisdiction of the International Criminal Court over Nationals of Non-Parties: Legal Basis and Limits, 1 J. Int’l Crim. Just. (2003) 618.

  57. 56.

    Shany, supra note 45, at 11–14. But see Al Haq, Position paper on issues arising from the PA submission of a Declaration to the Prosecutor of the ICC under article 12(3) of the Rome Statute, 14 December 2009, available online.

  58. 57.

    Al Haq, Position paper on issues arising from the PA submission of a Declaration to the Prosecutor of the ICC under article 12(3) of the Rome Statute, 14 December 2009, paragraphs 33–34, available online.

  59. 58.

    In this respect the view taken by the Arab League Committee is inconsistent, as it claims both that Israel continues to be the occupying power, and that the Palestinians have criminal jurisdiction.

  60. 59.

    See sources supra note 35; for a contrary view see Shany, Yuval, Faraway, So Close: the Legal Status of Gaza after Israel’s Disengagement, 8 Yearbook of International Humanitarian Law (2005) 369.

  61. 60.

    Opinion no 1 § 1 of the Badinter Arbitration Committee, appendix to A. Pellet, The Opinions of the Badinter Arbitration Committee: A Second Breath for the Self-Determination of Peoples, (1992) 3 EJIL 178; T.D. Grant, The Recognition of States: Law and Practice in Debate and Evolution (Westport: Praeger, 1999) ch 2.

  62. 61.

    E.g. the cases of Croatia and Bosnia and Herzegovina, which were recognized while not yet in effective control over most of their territories. Roland Rich, Recognition of States: The Collapse of Yugoslavia and the Soviet Union, 4 EJIL (1993) 36, at 49; Danilo Türk, Recognition of States: A Comment, 4 EJIL (1993) 66, at 69.

  63. 62.

    For an exceptional recognition by an international organization independently of its member states, see Statement on Membership of the Republic of Kosovo in the IMF, Press Release no 08/179, 15 July 2008, available online.

  64. 63.

    On 21 June 1989 the Swiss Federal Department of Foreign Affairs received a letter from the Permanent Observer of Palestine to the UN informing the Swiss Federal Council that the Executive Committee of the Palestine Liberation Organization, entrusted with the functions of the Government of the State of Palestine by decision of the Palestine National Council, decided, on 4 May 1989, to adhere to the Four Geneva Conventions of 12 August 1949 and the two Protocols additional thereto.

    The depositary Swiss Federal Council informed the states parties that it was not in a position to decide whether the letter constituted an instrument of accession, “due to the uncertainty within the international community as to the existence or nonexistence of a State of Palestine”. Embassy of Switzerland, Note of Information sent to States parties to the Convention and Protocol, 13 September 1989.

  65. 64.

    According to a news report, the Palestinians are pursuing membership in the ICC through accession to the Statute. Ma’an News Agency, Justice minister: PA prepping for ICC membership, 17 October 2009, available online, last visited 15 December 2009. The report suggests that these efforts are made vis-à-vis the ICC Prosecutor. However, accession is done by deposition of the relevant instrument with the UN Secretary General, who is the ICC Statute’s depositary.

  66. 65.

    See Loizidou v Turkey (Merits), ECtHR, 18 December 1996, Reports of Judgments & Decisions 1996-VI 2216, § 23.

  67. 66.

    Arts 15(1), 15(4) and 42 ICCSt.

  68. 67.

    Art. 53(3) ICCSt.

  69. 68.

    Art. 112(2) ICCSt. A notable exception is the transitional power in Art. 112(2)(a) ICCSt. to adopt recommendations of the Preparatory Commission, among which are proposals on aggression.

  70. 69.

    Mahnoush H. Arsanjani, The Rome Statute of the International Criminal Court; Exceptions to the Jurisdiction in M. Politi & G. Nesi (eds), The Rome Statute of the International Criminal Court, a challenge to impunity, (Aldershot: Ashgate, 2001) 49, at 50.

  71. 70.

    A. Bos, From the International Law Commission to the Rome Conference (1994—1998), in Cassese, Gaeta & Jones, supra note 55, at 308.

  72. 71.

    Art. 119(2) ICCSt.

  73. 72.

    A. Pellet, Les effets de la reconnaissance par la Palestine de la compétence de la CPI, 18 February 2010, paragraphs 5–15, available online.

  74. 73.

    Norwich et al. v. the Palestinian Authority and Yasser Arafat, Jerusalem District Court Civil Case 2538/00, Judgment, 30 March 2003, § 11, confirmed in Palestinian Authority v. Dayan et al., Request of Right to Appeal 4060/03, High Court of Justice, Judgment, 17 July 2007, § 4; Elon Moreh College v State of Israel, Jerusalem District Court, Judgment on Civil Case Request 1008/06, 24 June 2006, § 12, reversed by Elon Moreh College v State of Israel et al., Judgment on Civil Appeal 5093/06, 6 August 2008.

  75. 74.

    M.R Brubacher, Prosecutorial Discretion within the International Criminal Court, 2 J. Int’l Crim. Just. (2004) 71, at 83.

  76. 75.

    E.g. Art. 305(1) of United Nations Convention on the Law of the Sea, Art. XII(1) of the Agreement establishing the WTO.

  77. 76.

    Decision on the Defence Motion for Interlocutory Appeal on Jurisdiction, Tadić (IT-94-1), Trial Chamber, 2 October 1995, § 70 (“an armed conflict exists whenever there is a resort to armed force between States or protracted armed violence between governmental authorities and organized armed groups or between such groups within a State”).

  78. 77.

    Examples of exceptions were ‘A’ Mandated territories which were treated as states for the purpose of nationality but were much less certainly states for other purposes. The Free City of Danzig was a state for the purposes of Art. 71(2) of the Rules of the Permanent Court, but whether it was a state for all purposes has been doubted, Crawford, supra note 52, at 31.

  79. 78.

    The same scenario would be applicable to South Ossetia and Georgia. Even if jurisdiction is based on the territorial nexus with a member state, Serbia, the problem may arise with respect to the decision on admissibility, when the question might arise whether “[t]he case is being investigated or prosecuted by a State which has jurisdiction over it” (Art. 17(1)(a) ICCSt.).

  80. 79.

    GA Res. 34/37, 21 November 1979, § 7.

  81. 80.

    The SADR claims to be a state. For present purposes, it is taken as an example of a non-state entity in effective control over territory and population.

  82. 81.

    The ICJ found that at the time of Spanish colonisation, neither Morocco’s nor Mauritania’s ties to Western Sahara were of territorial sovereignty. Western Sahara Advisory Opinion, International Court of Justice, Reports (1975) 12, § 162; GA Res. 63/15, 18 December 2008, preamble.

  83. 82.

    The seminal commentary on the ICC Statute does not even mention the issue of non-state entities in the context of Art. 12, Williams and Schabas, supra note 23.

  84. 83.

    ASP Newsletter Special Edition #1 (May 2009), at 8, available online, last visited 15 December 2009.

  85. 84.

    Preamble, ICCSt.

  86. Suggested Citation for this Comment:

    Yaël Ronen, Admission of the Palestinian Declaration Would Require the ICC Prosecutor to Engage in Questions of Statehood and Recognition, Either Generally or for the Purposes of the ICC Statute, ICC Forum (Sep. 1, 2010), available at https://iccforum.com/gaza#Ronen.

    Suggested Citation for this Issue Generally:

    Should the ICC Investigate War Crimes in Gaza?, ICC Forum (Sep. 1, 2010), available at https://iccforum.com/gaza.